Passpoint Questions

Réussis tes devoirs et examens dès maintenant avec Quizwiz!

Which classification of drugs is the most potentially fatal if a client takes an overdose? Antihistamines Dopaminergics Phenothiazine antipsychotics Tricyclic antidepressants

Tricyclic antidepressants can create fatal cardiac arrhythmias. Overdose of antihistamines, dopaminergics, or phenothiazine antipsychotics is rarely fatal.

The nurse is teaching a group of pregnant women about hormonal changes during pregnancy. The nurse recognizes that teaching was successful when the women identify which hormone as causing amenorrhea? 1. Progesterone 2. Estrogen 3. Follicle-stimulating hormone (FSH) 4. Human chorionic gonadotropin (hCG)

1. Correct: Progesterone causes amenorrhea. 2. Incorrect: Estrogen renders the female genital tract suitable for fertilization. 3. Incorrect: This stimulates the growth of the graafian follicle in the ovary. 4. Incorrect: This is the hormone present in urine for pregnancy test

A client has recently been diagnosed with systemic scleroderma. Which of the following client complaints would be of most concern to the homecare nurse? 1. "I feel like food gets stuck in my throat when I eat." 2. "I have a hard time brushing my teeth properly." 3. "My fingers burn when I go outside in the winter." 4. "I get short of breath whenever I exercise."

1. Correct: Scleroderma is an autoimmune disorder characterized by the excess production of collagen and hardening of tissues. In systemic scleroderma, body organs lose the ability to function as the disease progresses. When parts of the digestive system build up collagen, clients experience frequent acid reflex, constipation, and difficulty swallowing. The nurse would be most concerned about aspiration during or after meals. 2. Incorrect: Because facial skin tightens, clients have difficulty opening the mouth completely. It becomes challenging to properly brush teeth or perform personal mouth care, increasing the likelihood of tooth decay. However, this would not be the biggest concern to the nurse at this time. 3. Incorrect: It is common for clients with one autoimmune disorder to develop other disorders. These symptoms indicate Raynaud's phenomenon, which is often reported by scleroderma clients. Advance of the disease triggers skin on the hands to become tight, stiff, and slightly shiny. The client begins to experience severe pain when fingers are exposed to the cold. Fingertips start out white in color, progressively turning red until re-warmed. Impaired circulation and pain are certainly an area of concern but not the most immediate worry to the nurse at this time. 4. Incorrect: Clients with scleroderma develop scarring ("fibrosis") of lung tissue, decreasing respiratory capabilities and eventually leading to pulmonary hypertension. This client reports shortness of breath just during exercise, indicating that simple daily activities are still achievable. It is obvious the disease has not yet progressed enough to impact ADL's, and therefore this is not the most concerning complaint at this time.

Which nursing task would be appropriate to delegate to an LPN/VN? 1. Obtain a wound culture from a client. 2. Administer regular insulin IV to a client in diabetic ketoacidosis. 3. Monitor a client's closed drainage unit (CDU) for tidaling. 4. Assess a client for tactile fremitus.

1. Correct: The LPN/VN has the knowledge and skill to obtain a wound culture. This is within the scope of practice for the LPN/VN. 2. Incorrect: The LPN/VN cannot administer IV medications to an unstable client. This client needs the RN for close monitoring. 3. Incorrect: The LPN/VN cannot monitor a chest tube and closed drainage unit. This is an RN task. 4. Incorrect: Assessment is the task of an RN. LPN/VN's can collect data, observe, and monitor the client.

The nurse is preparing to educate a client diagnosed with essential hypertension on how to decrease the risk of developing complications. What topics should the nurse include? 1. Following the DASH dietary plan. 2. Use of blood pressure monitoring device. 3. Diaphragmatic breathing exercises. 4. Brisk walking for 30 minutes 3-4 times/week. 5. Reduce sodium intake to less than 2700 mg/day.

1., 2., 3., & 4. Correct: The DASH Eating Plan is recommended for clients who have hypertension. It is high in vegetables, fruits, low-fat dairy products, whole grains, poultry, fish, beans, and nuts and is low in sweets, sugar-sweetened beverages, and red meats. Home blood pressure monitoring can help the client keep closer tabs on their blood pressure, show if medication is working, and even alert the client and primary healthcare provider to potential complications. However, home blood pressure monitoring isn't a substitute for follow-up visits. Reducing stress as much as possible helps to decrease blood pressure. Healthy coping techniques, such as muscle relaxation, deep breathing or meditation are good options. Getting regular physical activity and plenty of sleep can help, too. Regular physical activity can lower blood pressure, manage stress, reduce the risk of several health problems and keep weight under control. 5. Incorrect: A limit of 1500 mg of sodium per day is preferred on the low sodium DASH diet. On the standard DASH diet 2,300 mg of sodium are allowed each day.

The charge nurse has received word that a mass casualty has occurred and beds are needed in the hospital. This will require discharging some current clients. Which client would be appropriate to seek permission from the healthcare provider to be discharged? 1. Client admitted with chest pain and has an elevated Troponin level. 2. Client with blood glucose of 500 mg/dL and pH of 7.3 receiving IV insulin. 3. Client admitted with hemothorax but no chest tube drainage in last 14 hours. 4. Client who underwent a laminectomy for spinal stenosis 12 hours earlier. 5. Elderly client who fell and is developing increased confusion.

3., and 4. Correct: These clients are the ones that would be considered the most stable and therefore, could be safely discharged. The client who had the hemothorax and has not had any drainage for 14 hours indicates that the hemothorax has resolved. The chest tube could be safely removed for the client to be discharged. Clients who have laminectomies often are released home the same day as the surgery, if there are no complications and the condition is stable. This may seem early to discharge a client who had back surgery, but the clients who need to be admitted would be considered unstable and would be a priority over this client. 1. Incorrect: This client would be considered unstable and therefore not a candidate for discharge. This client may be having a MI as indicated by the presence of chest pains and the elevated Troponin level. Remember, Troponin is one of the most specific cardiac biomarkers for indicating myocardial damage. 2. Incorrect: This client is in diabetic ketoacidosis (DKA) and considered unstable. This client is in need of continued IV insulin and careful monitoring. Metabolic acidosis is an unstable condition. 5. Incorrect: Although you may think that it is normal for the elderly to have some confusion, this client may have a cerebral bleed following the fall and is considered unstable. If the client is found to not have a bleed or other complication from the fall and is determined to be stable, this would be the next client who would be considered for discharge.

What foods should the nurse inform the client to avoid for three days prior to a guaiac test? 1. Chicken 2. Carrots 3. Apple 4. Raw broccoli 5. Steak 6. Turnip greens

4., 5., & 6. Correct: Foods that affect this test include raw broccoli, red meats such as steak, turnip greens, cantaloupe, radish, and horseradish. All of these could cause a false positive reading for the guaiac test. 1. Incorrect: Red meats such as steak should be avoided, but chicken is okay. 2. Incorrect: Carrots are not prohibited and will not affect the results of the test. 3. Incorrect: The client can eat apples with no effect on the test results.

The health care provider (HCP) has prescribed pseudoephedrine. The nurse should instruct the client about which possible adverse effect of this drug? constipation bradycardia diplopia restlessness

Adverse effects of pseudoephedrine are experienced primarily in the cardiovascular system and through sympathetic effects on the central nervous system (CNS). The most common CNS adverse effects include restlessness, dizziness, tension, anxiety, insomnia, and weakness. Common cardiovascular adverse effects include tachycardia, hypertension, palpitations, and arrhythmias. Constipation and diplopia are not adverse effects of pseudoephedrine. Tachycardia, not bradycardia, is an adverse effect of pseudoephedrine.

When teaching parents about fifth disease (erythema infectiosum) and its transmission, the nurse should provide which information? Fifth disease is transmitted by respiratory secretions. Fifth disease has an unknown transmission mode. Fifth disease is transmitted by respiratory secretions, stool, and urine. Fifth disease is transmitted by stool.

Fifth disease is transmitted by respiratory secretions. The transmission mode for roseola is unknown. Rubella is transmitted by respiratory secretions, stool, and urine. Intestinal parasitic conditions, such as giardiasis and pinworm infection, are transmitted by stool.

The nurse is caring for a client with chronic renal failure who receives dialysis treatment. Which findings would indicate to the nurse that the client's AV shunt is patent? 1. A bruit is heard with a stethoscope. 2. A thrill is felt on palpation. 3. There is a blood return on the venous side of the shunt. 4. Urine output greater than 30 mL/hr. 5. There is a strong radial pulse in the arm with the AV shunt.

1, & 2. Correct: AV shunts should have the presence of a bruit and a thrill which indicates patency. 3. Incorrect: IV sticks should not be performed on the shunt or the extremity where the shunt is placed except for initiating dialysis. 4. Incorrect: This is not related to patency of AV shunt. This would be related to assessing the patency of an indwelling catheter. 5. Incorrect: Radial pulse does not determine patency of AV shunt. Only the confirmation of a bruit and a thrill ensure patency.

The nurse at the wellness clinic is teaching a client newly diagnosed with insulin-dependent diabetes mellitus. The client asks about beginning an exercise program. The nurse bases the response on the fact that exercise has what effect on the body? 1. Lowers the blood glucose 2. Provides more energy 3. Increases insulin need 4. Reverses complications of diabetes 5. Increases the workload of the liver

1. & 2. Correct: In the presence of adequate insulin, exercise lowers the blood glucose. Exercise releases endorphins, providing the client with increased energy and feelings of well-being. 3. Incorrect: Exercise does not require the need; for the increased production of insulin. 4. Incorrect: Exercise does not reverse complications. Exercise helps prevent microvascular and macrovascular changes/complications. 5. Incorrect: Exercise does not increase the workload of the liver.

The nurse is caring for a burn victim with a skin graft to the hand. The area is pale and mottled but has good capillary refill. What is the nurse's best action at this time? 1. Warm the room. 2. Submerge the hand in warm water. 3. Order a K pad and apply to hand. 4. Have the client exercise the fingers to increase blood flow.

1. Correct: When caring for clients with skin grafts, we want good circulation, so warm that room up. 2. Incorrect: This will not improve circulation and can lead to infection. 3. Incorrect: This will not improve circulation. Someone who has a skin graft doesn't have good sensation so there is risk of another burn to the graft with this. 4. Incorrect: Working those stiff, cold fingers will further imbalance the oxygen supply. This will not help, particularly if the environment remains cool.

The charge nurse has assigned four clients due for a morning assessment. The nurse knows what client should be assessed first? 1. A client with diabetis admitted for debridement of a foot ulcer. 2. A client with epilepsy reporting an odd smell in the room. 3. A client with exacerbation of COPD reporting dyspnea. 4. An adolescent client post appendectomy reporting pain.

2. CORRECT: The client is potentially experiencing symptoms of an impending seizure, which can include seeing halos around lights or detecting odd smells. The nurse should immediately assess this client, implement seizure precautions and remain with client for safety. 1. INCORRECT: Although the vascular status of the foot will need to be assessed, there is no indication if the debridement has been completed yet. This client is not the nurse's first priority. 3. INCORRECT: Clients with COPD are always short of breath and dyspnea is an expected finding during an exacerbation. The client will need to be assessed, but there is no specific indication the respiratory status is presently compromised. 4. INCORRECT: There is no information regarding how recent was the surgery or the degree of pain being experienced. Post-surgical pain is expected and without further parameters, no determination can be made regarding this client. The nurse has another priority.

Which signs/symptoms should the nurse assess for when caring for a client diagnosed with bulimia nervosa? 1. Increased thirst 2. Muscle cramps 3. Blurred vision 4. Tingling of lips 5. Constipation

2., 4., 5. Correct: The typical abnormalities associated with bulimia are hypokalemia and metabolic alkalosis because of the binging and purging process. This leads to muscle cramps, weakness, fatigue, constipation, and arrhythmias are all symptoms of this electrolyte and acid-base imbalance. Hypokalemia leads to metabolic alkalosis. 1. Incorrect: Increased thirst is a sign of hyperglycemia and would not be the concern with someone that is purging. This client would be more likely to be hypoglycemic instead. 3. Incorrect: Blurred vision is a sign of hyperglycemia because of the effect of too much glucose in the small vessels of the eye. Microvascular damage is one of the biggest concerns with hyperglycemia; the bulemic client would be hypoglycemic.

What should the nurse expect to include as a priority in the plan of care for a client with delirium based on the nurse's understanding of the disturbances in orientation associated with this disorder? identifying self and making sure that the nurse has the client's attention eliminating the client's napping in the daytime as much as possible engaging the client in reminiscing with relatives or visitors avoiding arguing with a suspicious client about his perceptions of reality

Identifying oneself and making sure that the nurse has the client's attention addresses the difficulties with focusing, orientation, and maintaining attention. Eliminating daytime napping is unrealistic until the cause of the delirium is determined and the client's ability to focus and maintain attention improves. Engaging the client in reminiscing and avoiding arguing are also unrealistic at this time.

A primipara at 36 weeks gestation is seen in the OB/GYN clinic. Which sign/symptom should the nurse immediately report to the primary healthcare provider? 1. Puffy hands and face 2. Reports indigestion 3. Pedal edema 4. Trace proteinurea

1. Correct: Facial and upper extremity edema can be a sign of pre-eclampsia, which can endanger both the mother and fetus. Preeclampsia is a pregnancy complication characterized by high blood pressure and signs of damage to another organ system, often the kidneys. Preeclampsia usually begins after 20 weeks of pregnancy in a woman whose blood pressure had been normal. Even a slight rise in blood pressure may be a sign of preeclampsia. Left untreated, preeclampsia can lead to serious, even fatal, complications. Signs and symptoms of preeclampsia include hypertension and may include: Proteinuria; Severe headaches; Changes in vision; Upper abdominal pain; Nausea or vomiting; Decreased urine output; Thrombocytopenia; Impaired liver function; Shortness of breath; Sudden weight gain, and edema, particularly in face and hands. 2. Incorrect: Indigestion should be assessed for severity, but it is a common symptom in 3rd trimester of pregnancy. 3. Incorrect: Pedal edema should be assessed but is common in 3rd trimester of pregnancy. 4. Incorrect: Trace proteinurea is a benign sign in 3rd trimester of pregnancy.

A child is admitted to the hospital with a temp of 102.2°F ( 39.0°C), lethargic, and no urinary output in 6 hours. Which prescription would be priority for the nurse to initiate for this child? 1. Blood cultures times two 2. Ceftriaxone 250 mg IV every 12 hours 3. Start IV & monitor site. 4. 1/2 normal saline at 40 mL/h

1. Correct: Immediate blood cultures should be obtained on this child, as sepsis is suspected with any temperature this high. The nurse would also need to get diagnostics before treatment is initiated so that correct interventions are prescribed. 2. Incorrect: The ceftriaxone is administered after the appropriate IV has been initiated. This would be the last intervention to be initiated. 3. Incorrect: The IV can be started at any point, but should be done after the cultures so the blood sample would not be affected in anyway. 4. Incorrect: Fluids will be started after the cultures are obtained and after the IV is started so as not to alter the results of the blood work and ensure correct treatment.

The nurse in the clinic would recognize which client statement as most indicative of gallbladder disease? 1. "Yesterday, when I ate a hamburger and french fries, my belly really hurt." 2. "I have been gaining a lot of weight lately." 3. "My stools are darker. Sometimes they are even black." 4. "When I start hurting, it helps if I drink milk or have a small snack."

1. Correct: The gallbladder assists in digestion of fat. When foods high in fat are ingested, bile is released from the gallbladder to assist in digestion. If gallstones are formed in the gallbladder or are blocking the outlet to the gallbladder, the client may experience epigastric discomfort after a meal high in fat. 2. Incorrect: Weight gain is not associated with gallstones. 3. Incorrect: Black stools indicate blood in the stool and should be further investigated. Black stools are not associated with gallstones. 4. Incorrect: When the symptom of drinking milk or having a small snack relieves the abdominal pain, a duodenal ulcer may be a possible diagnosis.

A client has a prescription for digoxin 0.125 mg IV push every morning. Prior to administering digoxin, the nurse notes that the digoxin level drawn this morning was 0.9 ng/mL. Which action would be most important for the nurse to take? 1. Administer the digoxin. 2. Hold the digoxin. 3. Notify the primary healthcare provider. 4. Repeat the digoxin level.

1. Correct: This is a normal digoxin level. The nurse would administer the prescribed digoxin. The therapeutic serum levels of digoxin range from 0.5 to 2 ng/mL. 2. Incorrect: This is a normal digoxin level. The nurse would administer the prescribed digoxin. 3. Incorrect: There is no need to notify the primary healthcare provider of a normal digoxin level. 4. Incorrect: There is no need to repeat a normal laboratory value.

The nurse is developing a teaching plan covering emergency responses to smallpox. This presentation will be used with newly hired hospital employees. What information is essential for the presentation? 1. People may be exposed to smallpox but not get the disease. 2. People may contract the disease by handling contaminated clothing or bedding. 3. Smallpox is fatal is about 50% of cases. 4. Smallpox victims are contagious for two weeks. 5. Smallpox victims are isolated from others.

1., 2. & 5. Correct: Not everyone who is exposed will develop the disease. Handling contaminated bedding or clothing is one way to contract the illness. If a person comes in contact with the wound discharge, infection may occur. Persons must be isolated for approximately 18 days after an exposure, and persons with smallpox should be isolated until the last scab falls off. 3. Incorrect: Smallpox is fatal in 10 - 30% of victims. 4. Incorrect: Smallpox is considered contagious until the last scab falls off. There is no specific time frame to determine this and is different for each individual.

The nurse has determined that a bedridden client diagnosed with a stroke is at risk for venous thromboembolism (VTE). What interventions should the nurse initiate? 1. Measure the calf and thigh daily. 2. Apply sequential compression device to legs. 3. Position paralyzed leg with each distal joint higher than the proximal joint. 4. Place a trochanter roll at the hip. 5. Perform passive range of motion exercises once daily. 6. Monitor for pain by assessing Homan's sign.

1., 2., & 3. Correct: Assessment for VTEs is accomplished by measuring the calf and thigh daily, observing swelling, noting unusual warmth of the leg, and asking the client about pain in the calf. Prevention of VTEs include the use of sequential compression devices for bedridden clients. This device helps promote venous return. Positioning the paralyzed leg with each distal joint higher than the proximal joint will prevent dependent edema. 4. Incorrect: A trochanter roll does not prevent VTEs. They are used for the prevention of external hip rotation. 5. Incorrect: Passive range of motion exercises should be done several times a day to promote venous return and muscle tone. Once a day is not adequate. 6. Incorrect: Routinely checking the Homan's sign can actually cause a clot to dislodge. The nurse should not perform this procedure.

A client is admitted to the Labor & Delivery Unit with severe preeclampsia. Which nursing intervention does the nurse include in the plan of care for this client? 1. Monitor for headache. 2. Place client in left recumbent position. 3. Insert indwelling urinary catheter. 4. Administer propranolol for BP > 100 diastolic. 5. Initiate external fetal heart monitoring.

1., 2., 3., & 5. Correct: Headache is a sign of increasing BP and increasing ICP. The left recumbent position moves the fetus off the mom's aorta and will help decrease the BP. This client needs to have UOP closely monitored because of the fluid volume excess (FVE), so an indwelling urinary catheter is needed. The fetus needs to be monitored for complications, and the fetal heart rate (FHR) should be 120-160/minute so close monitoring is required. 4. Incorrect: Propranolol, a beta blocker, is not given during pregnancy as it decreases HR and the amount of blood pumped by the heart. This can cause fetal bradycardia, decreased cardiac output, and potential for fetal demis

Which tasks could the nurse working on a cardiac unit delegate to an unlicensed assistive personnel (UAP)? 1. Bathe the client who is on telemetry. 2. Apply cardiac leads and connect a client to a cardiac monitor. 3. Help position a client for a portable chest x-ray. 4. Feed a client who is dysphagic. 5. Collect a stool specimen.

1., 2., 3., & 5. Correct: Remember the RN cannot delegate assessment, teaching, evaluation, medications, or an unstable client to the UAP. The UAP could bathe the client who is on telemetry. This is an appropriate assignment. The UAP can apply cardiac leads and connect the client to a cardiac monitor. The UAP can assist with helping the client sit up for a portable chest x-ray as long as the UAP is not pregnant and wears a shield. The UAP can collect specimens, such as a stool specimen. 4. Incorrect: This client has difficulty swallowing and is at risk for choking making the client unstable. Therefore, the nurse should not allow the UAP to feed this client.

The nurse is planning care for a client who has a fractured hip. Which nursing interventions should the nurse plan to use for impaired physical mobility? 1. Turn every two hours 2. Place a pillow between legs when turning 3. Sit in a chair three times per day 4. Encourage fluid intake 5. Encourage ankle and foot exercises

1., 2., 4. & 5. Correct: The client must be turned every two hours. You may not be able to turn the client totally on the side of the fracture, but you must relieve pressure points. Place pillow between legs to keep affected leg in abducted position. Encourage fluid intake and ankle and foot exercises to prevent deep vein thrombosis (DVT). 3. Incorrect: The client has a fractured hip that has not been surgically fixed. Sitting up in a chair could do more injury and cause more pain.

A nurse suspects that a client admitted to the emergency department is in diabetic ketoacidosis. What data would lead the nurse to this conclusion? 1. Dry mucous membranes 2. Fruity-smelling breath 3. Biot's respirations 4. Glycosuria 5. Client report of abdominal pain

1., 2., 4., & 5. Correct: The client with diabetic ketoacidosis will have signs of dehydration due to polyuria and includes dry mucous membranes. Fruity breath odor is from the acetone that occurs with breakdown of fats and formation of ketones, which are acids.. With DKA, the client would be spilling glucose into the urine. Vomiting and abdominal pain are frequently the presenting symptoms of DKA. 3. Incorrect: The client will have Kussmaul respirations. Biot's respiration is a respiratory pattern characterized by periods of rapid respirations, then apnea periods. These are not the type of respirations that occur with diabetic ketoacidosis (metabolic acidosis).

After determining that a client diagnosed with a stroke has adequate swallowing ability, the nurse develops interventions to safely provide oral feedings to the client. What interventions should the nurse include in this plan of care? 1. Provide mouth care prior to feeding. 2. Flex head forward for eating. 3. Have dietary puree foods. 4. Use crushed ice as a stimulant for swallowing. 5. Offer thickened liquids to drink. 6. Position client in semi fowler's position after feeding.

1., 2., 4., & 5. Correct: These interventions will stimulate sensory awareness, salivation, swallowing, and decrease the risk of aspiration. 3. Incorrect: Pureed foods are not usually the best choice because they are often bland and too smooth making it difficult to swallow. 6. Incorrect: The client should remain in a high Fowler's position, preferably in a chair with the head flexed forward, for feeding and for 30 minutes afterward.

Which manifestations, if noted in a pregnant client, would the nurse need to report to the primary healthcare provider? 1. Calf muscle irritability 2. Facial edema 3. Pressure on the bladder 4. Blurry vision 5. Hemoglobin of 11 mg/dL 6. Epigastric pain

1., 2., 4., & 6. Correct: These are danger signs/symptoms of pregnancy and need further investigation by the primary HCP. These signs could indicate preeclampsia, fluid and electrolyte disturbances, and other high risk complications during pregnancy. 3. Incorrect: As the baby gets bigger, it pushes on the bladder, causing pressure, so this is an expected symptom in pregnancy. 5. Incorrect: This is normal for the pregnant client and within the normal range for the female client.

Which task would be appropriate for the charge nurse to assign to a LPN/VN? 1. Collect data on a new client admit. 2. Administer morphine IVP to a two day post-op client. 3. Bolus feeding a client who has a gastrostomy tube. 4. Reinserting a nasogastric tube (NG) that a client accidentally pulled out. 5. Monitor patient control analgesic (PCA) pump pain medication being delivered to a client.

1., 3., 4., & 5. Correct: All of these tasks are appropriate and within the scope of practice for the LPN/VN. The LPN/VN can collect data on a new admit, and the RN would verify and co-sign to complete the assessment. Bolus feeding by way of a gastrostomy tube and reinserting a nasogastric tube would be appropriate assignments for the LPN/VN also. A LPN/VN can monitor the PCA pain medication but cannot initiate or administer the medication. 2. Incorrect: Administering morphine IVP is out of the scope of practice for the LPN/VN since it is a complex, high risk IV push medication and has the potential to depress the client's respiratory rate.

A client has been admitted to the medical unit after sustaining a stroke. The admitting nurse initiates a nursing diagnosis of unilateral neglect related to a decrease in visual field and hemianopia from cerebrovascular problems as evidenced by consistent inattention to stimuli on the affected side. What nursing interventions should the nurse initiate for this client? 1. Instruct client to scan from left to right to visualize the entire environment. 2. Encourage client to practice exercises independently. 3. Position bed in room so that individuals approach the client on the unaffected side. 4. Apply splints to achieve stability of affected joints. 5. Touch unaffected shoulder when initiating conversation with client. 6. Position personal items within view on the unaffected side.

1., 3., 5., & 6. Correct: Instructing the client to scan from left to right will help the client to visualize the entire environment. The client has to be reminded to do this since only one side of the client's visual field is working. By positioning the bed so that individuals approach the client from the unaffected side and by touching the client on the unaffected shoulder, the client is not surprised or frightened when realizing someone is in the room. Placing personal items where the client can see them will allow the client to use the material. Then gradually move personal items and activity to the affected side as the client demonstrates an ability to compensate for neglect. 2. Incorrect: Practicing exercises independently focuses on impaired physical mobility rather than unilateral neglect. 4. Incorrect: Applying splints to affected joints focuses on impaired physical mobility rather than unilateral neglect.

The nurse is providing teaching for a client who is being scheduled for outpatient 24 hour electrocardiogram monitoring using a Holter monitor. What should the nurse tell the client to avoid while monitoring is in progress? 1. Taking a shower or bath 2. Performing daily exercises 3. Working around high voltage equipment 4. Being screened at airport security 5. Eating foods that are sources of potassium

1., 3., and 4. Correct: The nurse should teach this client to continue the usual activities while wearing the monitor with a few exceptions. The monitor should be kept dry to ensure that it functions properly. The client should avoid taking a shower or bath or swimming while wearing the monitor. The electrodes could also become detached from the skin if they get wet, which would also interfere with the accuracy of the reading. The client should be advised to not work around high voltage equipment because areas of high voltage can interfere with the function of the electrocardiogram monitoring. In addition, magnetic fields, such as those used for airport screenings, can interfere with the function of the Holter monitor and should be avoided. 2. Incorrect: This client should be encouraged to continue regular routine unless otherwise directed by the primary healthcare provider. The client can perform the usual daily exercise, but should be advised to avoid activities that may cause excessive perspiration that could lead to the electrodes becoming loosened from the skin. 3. Incorrect: There are generally no dietary restrictions while wearing the Holter monitor unless otherwise prescribed by the primary healthcare provider.

In which situations should the nurse notify the primary healthcare provider of a medication incident? 1. Every occurrence. 2. Client is harmed or dies. 3. Medication incident is a near miss. 4. Nurse administers an incorrect dosage. 5. Client questions the medication color.

2. & 4. Correct: The primary healthcare provider should be notified if harm is brought to the client or death occurs as a result of the medication incident. The primary healthcare provider should be notified if the nurse administers an incorrect dosage to the client, and an incident report needs to be completed in this situation. 1. Incorrect: The primary healthcare provider should be notified if harm is brought to the client but not for all events with medications. An incident report should be completed so the hospital can track incident patterns for quality improvement. 3. Incorrect: Near misses do not need to be reported to the primary healthcare provider. Following the rights of medication administration every time ensures medication error prevention. 5. Incorrect: The nurse should answer questions regarding medication color. Depending on the manufacturer, the shape and color of the medication can vary.

The nurse is teaching a group of clients about selective serotonin reuptake inhibitors (SSRI). Which comment by a client in the group indicates adequate understanding of the effects/side effects of the medications? 1. My weight may decrease while taking this drug. 2. I may expect increased sweating while taking this drug. 3. I may actually feel more depressed while taking this medication. 4. I should feel better within a couple of days after beginning the medication.

2. Correct. The drug causes temperature dysregulation, with increased sweating in some clients. 1. Incorrect. The medications may cause weight gain in some clients. 3. Incorrect. The client should have a lessening of depressive symptoms within a few weeks. This is one of the primary indications for taking this classification of medications. 4. Incorrect. The lag time for antidepressants to reach therapeutic effect is usually two to four weeks before the therapeutic effect is reached. The client's comment indicates lack of understanding of the medication effects and side effects

A client who has had a laparoscopic cholecystectomy develops pain in the left shoulder. Vital signs, laboratory studies, and an electrocardiogram are within normal limits. What does the nurse recognize as a contributing cause of the pain? 1. Surgical cannulation of the bile duct is causing spasm and pain. 2. Carbon dioxide used intraperitoneally is irritating the phrenic nerve. 3. Large abdominal retractors used in the procedure compressed a nerve. 4. Side lying position in the operating room generated pressure damage.

2. Correct: Phrenic nerve irritation can result in referred pain to the left shoulder. Carbon dioxide (CO2) is used to inflate the abdominal/chest wall during the procedure for better visualization of the internal organs. If the CO2 irritates the phrenic nerve, it radiates to the shoulder. 1. Incorrect: Surgical cannulation of the bile duct is not performed during a laparoscopic cholecystectomy. 3. Incorrect: Large abdominal retractors are not used during this procedure. This is done via a small incision to accommodate a scope. 4. Incorrect: The client is turned in several directions during the procedure to prevent damage to the abdominal viscera.

The following clients arrive to the emergency department (ED) at the same time. The triage nurse gives priority to which client? 1. A client with a possible fracture of the tibia 45 minutes ago. 2. A client with left hemiparesis and aphasia beginning 1 hour ago. 3. A client smelling of alcohol and reporting of severe abdominal pain. 4. A client involved in a motor vehicle accident (MVA) with a possible fractured pelvis.

2. Correct: The client who is started experiencing hemiparesis and aphasia 1 hour ago is likely having a stroke. The window for treatment with fibrolytics is 3 hours, thus taking priority over the other clients. Time is brain! 1. Incorrect: This client has a possible fracture of the tibia. This is not a large bone, which would be at risk for hemorrhage. Splinting and ice packs could be used until after seeing the client having a stroke. 3. Incorrect: With this client, you would worry about pancreatitis. This client needs to be seen soon but not prior to the client having a stroke. 4. Incorrect: The MVA client could have bleeding from a fractured pelvis. This client is high on the admit list, but after the client having a stroke.

The nurse sees that the new medication noted in a recent prescription is on the client's list of allergies. In the role of client advocate, what actions should the nurse take to ensure client safety? 1. Document the medication with times and doses to be given, then administer the medication as ordered. 2. Notify the primary healthcare provider immediately that the medication prescribed is on the client's list of medication allergies. 3. Stop the medication on the client's medication administration record. 4. Check the client's allergy band against the list of client allergies documented in the medical record. 5. Call the pharmacy to see if the medication needs to be changed.

2., 3. & 4. Correct: Administration of a medication that the client is allergic to could result in harm to the client. The primary healthcare provider should be notified immediately of a medication prescription that conflicts with the client's list of medication allergies. The medication should be discontinued on the medication administration record, and the client's allergy band checked against the list of allergies documented in the medication record for accuracy. All of these actions place the nurse in the role of client advocate and ensure the client's safety. 1. Incorrect: No, this medication could cause harm to the client. The client is allergic to this medication. 5. Incorrect: No, the primary healthcare provider, not the pharmacy, should be notified for medication changes. The primary healthcare provider is responsible for prescribing the medication.

What symptoms does the nurse expect to see in a client with bulimia nervosa? 1. Amenorrhea 2. Feelings of self-worth unduly influenced by weight 3. Recurrent episodes of binge eating 4. Recurrent inappropriate compensatory behavior to prevent weight gain 5. Lack of exercise

2., 3. & 4. Correct: Diagnostic criteria for bulimia nervosa are recurrent episodes of binge eating: recurrent inappropriate compensatory behavior to prevent weight gain such as laxative, diuretic, or enema use, induced vomiting, fasting, and excessive exercise; and feeling of self-worth unduly influenced by weight. Amenorrhea is found in anorexia nervosa. 1. Incorrect: Amenorrhea is found in anorexia nervosa. 5. Incorrect: Excessive exercise is found in bulimia nervosa as a means to compensate for the binge eating.

A client is diagnosed with new onset grand mal seizures. Which nursing interventions should the nurse implement for this client? 1. Have an unlicensed assisitve personnel stay with the client. 2. Pad the side rails with blankets. 3. Place the bed in low position. 4. Keep a padded tongue blade at the bedside. 5. Instruct client to call for help when ambulating.

2., 3., & 5. Correct: During a seizure these interventions will help to protect the client from injury. The client may strike the side rails. The bed should be placed in the low position in case the client falls out of the bed. The client would need assistance to the floor if a seizure starts while ambulating. 1. Incorrect: It is not necessary to have someone stay with this client at all times. Place a call light within reach, put the client close to the nurses' station, and pad the side rails. Have the client call for assistance to bathroom. Maintain bed rest until seizures are controlled or ambulate the client with assistance to protect from injury. 4. Incorrect: Do not place a padded tongue blade in a client's mouth during a seizure. The padded tongue blade could cause injury.

What preoperative education should the nurse provide to a client scheduled for a transsphenoidal hypophysectomy? 1. There will be a large dressing covering the forehead. 2. Avoid drinking from a straw. 3. The head of the bed will be elevated 30 degrees. 4. After surgery, your urinary output will be monitored. 5. Use a sponge when doing oral care.

2., 3., 4., & 5. Correct. Drinking from a straw can damage the surgical area, so the client should not use a straw until approved by the primary care provider. The client will return from surgery with the head of the bed elevated to about 30 degrees. This will allow for gravity to assist with draining of any cerebrospinal fluid. Surgery can damage the posterior lobe of the pituitary gland, which makes the client urinate frequently and feel thirsty. This is diabetes insipidus. If the client puts out too much urine, then fluid volume deficit or shock can occur. The incision is just above the gumline, so the client should not brush the front teeth. Oral care is best performed using a sponge until the incision heals. 1. Incorrect: Pituitary surgery can be removed through traditional microscopic or endoscopic technique. So, there is no dressing on the forehead.

What interventions should the nurse include when planning care for a client post heart transplant? 1. Place on airborne precautions. 2. Instruct visitors to wash hands prior to entering the room. 3. Maintain strict aseptic technique. 4. Initiate pulmonary hygiene measures. 5. Provide for early ambulation.

2., 3., 4., & 5. Correct: The transplant recipient is at high risk for infection due to the suppression of the body's normal defense mechanisms. All of these interventions decrease the incidence of the client developing an infection. The heart transplant client is prescribed medications to reduce the risk of organ rejection by inhibiting or suppressing the immune system. Handwashing is the main defense against infection. Pulmonary hygiene measures are are implemented to maintain open airways and prevent respiratory infections. The pulmonary measures can include oral hygiene, deep breathing exercises, mucus-controlling agents, and intermittent positive-pressure breathing. Pulmonary hygiene helps to decrease the development of pneumonia. Early ambulation helps increase general strength and lung expansion. Also ambulation increases circulation,peristalisis,and joint mobility, Emotionally ambulation improves self-esteem and feelings of independence. 1. Incorrect: The client needs to be protected from everyone else so a private room and protective isolation are needed. All persons entering the room must wash hands well and wear a mask and gloves. The client must wear a mask when leaving the room.

A client is hospitalized for chronic renal failure. The nurse will need to notify the primary healthcare provider concerning which findings? 1. Sodium 135 mEq/L 2. Potassium 5.8 mEq/L 3. BP 100/70 4. No weight loss 5. Ionized Calcium 4.0 mg/dL

2., 5. Correct: Normal K 3.5-5.0 mEq/L; Normal ionized serum Ca 4.5-5.5 mg/dL. The abnormal lab results need to be reported. 1. Incorrect: Normal sodium 135-145 mEq/L. 3. Incorrect: Hypertension is a potential complication of chronic renal failure. 4. Incorrect: Desired outcome: client exhibits no rapid increases or decreases in weight.

Which assignments would be most appropriate for the RN to delegate to an LPN/VN? 1. Six year old with new onset diabetes. 2. Ten year old with pneumonia admitted two days ago. 3. Three month old admitted with severe dehydration. 4. Four year old admitted for developmental studies. 5. Twelve year old with post op wound infection taking oral antibiotics.

2.,4. & 5. Correct: The best assignments for the LPN/VN would be the child with pneumonia admitted two days ago and the child admitted for developmental studies. The twelve year old with post op wound infection taking oral antibiotics is also stable. 1. Incorrect: The diabetic requires much teaching and supervision. This is an unstable client that should not be assigned to an LPN/VN. 3. Incorrect: The child with dehydration will require close intravenous fluid (IVF) monitoring, assessment and evaluation of condition. This client is unstable and should not be assigned to an LPN/VN.

A toddler with a malfunctioning ventriculoperitoneal (VP) shunt has returned from surgery following new shunt placement. Which post-op assessment finding should the nurse report to the primary healthcare provider immediately? 1. Blood pressure of 90/45 with pulse of 100 2. Urinary output of 30 mL over two hours 3. Sleeping soundly and difficult to arouse 4. Respirations deep and shallow at 20/min

3. CORRECT: Though the toddler is recovering from anesthesia, the nurse should be able to arouse and awaken the client, even expecting some crying. Difficulty arousing this client is one sign of increased intracranial pressure and should be reported immediately. 1. INCORRECT: These vital signs are well within normal limits for the toddler age-group, even post-op. No concerns for the nurse here. 2. INCORRECT: An output of 30 milliliters may seem a bit low, but the toddler is still recovering from surgery and anesthesia, with IV fluids still infusing to rehydrate. Additionally, two hours is not long enough to establish a consistent pattern. 4. INCORRECT: The respiratory rate is within normal limits for this client now. However, with potential changes in the neurological status of this client, the nurse would monitor for any decrease in respirations.

A client who underwent a laparoscopic cholecystectomy is being discharged from an outpatient surgical center. Which statement by the client shows the nurse that discharge teaching has been effective? 1. I will need to eat a low fat diet since I no longer have a gallbladder. 2. I can expect drainage from the incisions for a few days. 3. I may have some mild pain from the procedure. 4. I should plan to limit my activities and not return to work for several weeks.

3. Correct: After a laparoscopic procedure the client can expect to have some mild pain. Severe pain, however, would indicate a problem. 1. Incorrect: The client can resume their usual diet. The liver will produce enough bile to digest fats. The gallbladder stores bile. Without the gallbladder, the bile just drains from the liver. 2. Incorrect: The client should not have drainage from the incisions. There are 2-3 small incisions on the abdomen that do not normally have drainage. 4. Incorrect: The client can return to normal activities in 2 to 3 days. This is not considered a major surgical procedure with a large abdominal incision. Recover time is much shorter, allowing the client to return to normal activities sooner.

The primary healthcare provider instructs the nurse to place body tissue obtained from a biopsy into a container with formalin prior to sending it to pathology. The nurse has not handled formalin before. What would be the nurse's best action? 1. Call the pathology department for directions on formalin's use and precautions. 2. Look formalin up in the drug handbook 3. Read about formalin on the Material Safety Data Sheet (MSDS). 4. Explain to the primary healthcare provider that nurses are not allowed to use formalin.

3. Correct: All hazardous materials must have a MSDS, which includes the identity of the chemical, the physical and chemical characteristics, the physical and health hazards, primary routes of entry, exposure limits, precautions for safe handling, controls to limit exposure, emergency and first-aid procedures, and the name of the manufacturer or distributor. 1. Incorrect: The nurse should look at the MSDS, the best source of information. Calling another department does not ensure that the nurse will get as comprehensive information as the MSDS provides. 2. Incorrect: The drug handbook is for medication, not handling of hazardous material. 4. Incorrect: The nurse can place the biopsy into a container with formalin and is within the scope of practice for the nurse.

The nurse is caring for a client diagnosed with type 2 diabetes who was brought to the emergency department in an unresponsive state. A diagnosis of hyperglycemic hyperosmolar nonketotic syndrome (HHNS) is made. The nurse prepares for the administration of which initial therapy? 1. Oxygen by nasal cannula 2. Long-acting IV insulin 3. Normal saline 4. IV dextran

3. Correct: Clients in HHNS diurese due to a high glucose load in the vascular space. The client becomes severely volume depleted and is at risk for developing shock. Therapy is focused on combating shock. 1. Incorrect: Oxygen by nasal cannula is not the priority for this client. Don't pick oxygen as a priority every time. Oxygen does not fix the problem. The problem is shock. 2. Incorrect: The client will be given short-acting insulin. 4. Incorrect: Dextran is contraindicated as this will increase blood sugar even more.

The nurse is admitting a client with a fifteen year history of poorly controlled diabetes mellitus. During the initial assessment the client reports experiencing "numb feet." What nursing action takes priority? 1. Check blood glucose level. 2. Assess for proper shoe size. 3. Examine the client's feet for signs of injury. 4. Test sensory perception in the client's feet.

3. Correct: Clients with decreased peripheral sensation are at risk for injury to the extremity. They may sustain an injury and be unaware the injury has occurred. In addition to this, diabetics are at risk for poor wound healing (related to impaired circulation) and infection (related to elevated glucose levels). This is the assessment that should be performed first and takes priority. 1. Incorrect: Checking a fasting blood glucose level is important, but it is not the FIRST action to be taken. Checking the blood glucose level does not fix the problem. The problem is potential risk for injury. Assessing for injury is the priority answer. 2. Incorrect: Diabetics need well-fitting shoes, but this is not the priority answer. Check the client first. 4. Incorrect: Checking the sensation in the feet is not fixing the problem. It will be done later but risk for injury is the priority because the client has numbness of the feet.

A client's membranes spontaneously rupture at 10 cm dilation and +2 station. The nurse notes that the fluid is colored green. What client preparation is the priority nursing action? 1. Emergency cesarean delivery 2. Immediate high forceps delivery 3. Equipment for immediate suctioning of the newborn 4. Administration of IV oxytocin

3. Correct: Green stained fluid indicates fetal passage of meconium. The fetus must be suctioned by the healthcare provider when the head is still on the perineum and before the baby takes its first breath. This will remove any particulate matter from the meconium that may cause aspiration. 1. Incorrect: Delivery will probably occur soon and vaginal delivery is preferable to cesarean. This is an unrealistic and inappropriate action for this client. 2. Incorrect: High forceps are never indicated and would not provide safe delivery for the baby. The concern is the meconium stained fluid and potential aspiration for the baby. 4. Incorrect: The meconium passage is an indicator of fetal stress, and increased uterine contractions may stress the fetus further. This would not be safe for the baby or the mother at this stage of labor.

A client is returned to the surgical unit following gastric/esophageal repair of a hiatal hernia, with an IV, NG tube to suction, and an abdominal incision. To prevent disruption of the esophageal suture line, what is most important for the nurse to do? 1. Assess the wounds for drainage. 2. Give ice chips sparingly. 3. Maintain the patency of the NG tube. 4. Monitor for the return of peristalsis.

3. Correct: Maintain the patency of the NG tube. On ANY post-op client, the nurse is responsible for preventing disruption of the suture line. (Disrupture of any suture line, since disruption could be life-threatening.) The nurse is responsible for keeping the NGT patent to prevent accumulations of gastric secretions and blood in the stomach. Accumulation of fluid in the stomach can cause pressure on the suture line and places the client at risk for disruption of the suture line and hemorrhage. The nurse knows NEVER to allow pressure or stretching on suture lines. 1. Incorrect: Assessing the wound for drainage is important, but when there is something more life-threatening, that is the priority answer. Disrupting the sutures is more life-threatening. 2. Incorrect: This person is ABSOLUTELY NPO. Giving ice chips is contraindicated as it could disrupt the suture lines. 4. Incorrect: It is important to monitor for return of peristalsis, but this is not life-threatening.

A nurse is caring for a client on the second day after a thoracotomy. The client reports incisional pain. The nurse assesses the client and evaluates the vital signs. Based on the data documented in the chart, what action should the nurse take first? 1. Have client cough and deep breathe. 2. Administer acetaminophen for fever 3. Administer the prescribed analgesic 4. Assist the client to ambulate.

3. Correct: The client described in this question is post thoracotomy. With ANY post-op client, the number one concern, especially as a brand new nurse, is preventing pneumonia. A thoracotomy is very painful and the client is unlikely to breathe deep unless the pain is relieved. Temperature of 100º F/37.8ºC, HR 92, respirations 24, bilateral crackles (indicating atelectasis) all reveal this client is heading for pneumonia. 1. Incorrect: Coughing and deep breathing exercises are exactly what the client needs, but the client will not cough and deep breathe if it hurts. Give pain medication first. 2. Incorrect: Acetaminophen is not potent enough to relieve pain. The goal is to "fix the problem". The problem is that the client is not properly deep breathing due to pain. 4. Incorrect: Assisting the client to ambulate is a good idea, but the nurse has to fix the problem, and the problem is that the client is not deep breathing.

After artificial rupture of membranes (AROM), the baseline fetal heart rate tracking begins to show sharp decreases with a rapid recovery with and between contractions. Which of the following actions by the RN has priority? 1. Position the client on her left side 2. Increase the IV fluid rate 3. Place the client in the knee-chest position 4. Administer oxygen per tight face mask

3. Correct: The fetal heart pattern is that of repetitive deep variable decelerations. This pattern is likely due to a prolapsed umbilical cord after AROM. The priority intervention is to relieve the pressure on the cord from being trapped between the presenting part and the pelvis. This can be accomplished by manual pressure on the presenting part, placing the client in Trendelenburg position, or placing her in the knee-chest position. 1. Incorrect: This intervention will improve placental perfusion, but will not relieve compression of a prolapsed cord. If the cord is compressed, it doesn't matter how well perfused the placenta is because the oxygen cannot reach the baby. Late decels and low BP would be an indicator that we need to increase uterine perfusion by positioning on left side. 2. Incorrect: This intervention will not improve placental perfusion. IV fluids will not relieve compression of a prolapsed cord. 4. Incorrect: This intervention will improve placental perfusion, but will not relieve compression of a prolapsed cord. If the cord is compressed, it doesn't matter how well perfused the placenta is because the oxygen cannot reach the baby.

A newly admitted client tells the nurse, "I am hearing voices." Which response by the nurse is most appropriate? 1. Your head is turned to the side as if you are listening to voices. 2. I don't hear anyone but you speaking. 3. Tell me what the voices are saying to you. 4. Let's talk about your anxiety right now.

3. Correct: The nurse needs to know what the voices are saying to the client. This is the first thing the nurse would ask if the newly admitted client tells the nurse about hearing voices. The nurse does not know the client or the diagnosis that might be affiliated with this statement. 1. Incorrect: The client has already told the nurse about hearing voices. This also is non-therapeutic and negates the value of what the client is saying. 2. Incorrect: Upon admission, the nurse would not start out with this comment. This would come later. First the nurse needs to know what the voices are telling the client. 4. Incorrect: Again, this would come later after the nurse finds out what the voices are telling the client.

The nurse is evaluating dietary education provided to a client diagnosed with cholecystitis. The nurse determines that further teaching is necessary when the client chooses which meal to consume? 1. Cup of oatmeal, blueberries, soymilk 2. Whole grain pasta, marinara sauce, baked fish, coffee 3. Spaghetti with meat sauce, peas, garlic French bread, tea 4. Lentil soup, vegetable medley, fruit salad, water

3. Correct: This is not a good choose for this client. Meat is high fat. French bread with butter is low fiber and high fat. 1. Incorrect: This is a good meal choose when on a low fat, high fiber diet. Blueberries are high in fiber and all are low fat. 2. Incorrect: This is a good low fat, high fiber meal choose. Whole grain pasta is high in fiber and low in fat. Fish and marinara sauce are low in fat. 4. Incorrect: These are low fat, high fiber items to consume.

A small community has experienced a severe tornado that hit a shopping mall and caused extreme damage and suspected mass casualties and injuries. Which intervention takes priority? 1. Triage victims and tag according to injury. 2. Assess the immediate area for electrical wires on the ground and in vicinity of victims. 3. Activate the community emergency response team. 4. Begin attending to injuries as they are encountered.

3. Correct: With mass casualties, community response teams are needed. 1. Incorrect: This would be the third step. 2. Incorrect: This would be the second step so that further injuries are not encountered. 4. Incorrect: Triage must occur before treatment of anyone so that an accurate assessment of level of injuries can be made. With mass casualties, a color tag system is usually implemented.

An alert client presents to the emergency department with vomiting for 3 days and has been unable to keep food or fluids down for the last 24 hours. Which imbalances does the nurse suspect this client has? 1. Hypocalcemia 2. Hypermagnesemia 3. Hypokalemia 4. Metabolic alkalosis 5. Respiratory acidosis

3., & 4. Correct: Clients who vomit lose acid; therefore, they will have metabolic alkalosis. A client who is not eating and is vomiting will also lose potassium. Potassium is the electrolyte most significantly lost from the upper GI tract. 1. Incorrect: Calcium is not the electrolyte that is altered significantly with vomiting. It is primarily lost from intestinal elimination. 2. Incorrect: Actually magnesium has to be replaced daily just like potassium, so it will be low also. 5. Incorrect: The client is alert and should be breathing ok. The problem is not with the lungs.

Prior to removal of cataracts, the client is to receive eye drops in both eyes. The nurse knows what action takes priority? 1. Remove any exudate around eyes with warm water. 2. Instill exact number of drops into lower conjunctival sac. 3. Instruct client to look upward when drops are instilled. 4. Avoid dropping the medication directly on the cornea.

4. CORRECT: The most important safety consideration when instilling eye drops is to avoid dropping the medication directly onto the cornea. The extreme sensitivity of the cornea before, and after, eye surgery could cause serious eye problems if meds were dropped onto the cornea. 1. INCORRECT: It is important to clean away any exudate prior to instilling eye drops to maintain aseptic technique and decrease chance of infection. Though this is an important action, there is another task which takes priority. 2. INCORRECT: Instilling the exact number of drops is appropriate when implementing written prescriptions from the primary healthcare provider. This is an important nursing action but not the priority. 3. INCORRECT: Instructing the client to look upward helps prevent drops from running out of the eye but there is another issue more important.

A home health nurse visits a recently discharged client with right-sided paresis due to a stroke. The nurse discovers that the spouse has been feeding the client. What action should the nurse take? 1. Instruct the spouse to require the client to feed independently. 2. Suggest the spouse hire an aide to feed and bathe the client. 3. Advise the spouse to consider an extended care facility for the client. 4. Determine why the spouse is not encouraging self-care by the client.

4. Correct: Because family members are important in promoting client self-care and preventing further illness, it is important to include family members in the teaching plan for the client. In a family support model, the goal is client self-care activities through formal and informal support systems. 1. Incorrect: Simply instructing the spouse to require the client to perform self-care activities may result in affirmative verbal response from the spouse without actual follow-through after the home health nurse leaves. 2. Incorrect: Hiring others to perform care activities that the client can do independently does not contribute to the self-care model. 3. Incorrect: There are no indications provided in the stem that the client needs an extended care facility.

Which client should the nurse recognize as being at greatest risk for the development of cancer? 1. Smoker for 30 plus years 2. Body builder taking steroids and using tanning salons 3. Newborn with multiple birth defects 4. Older individual with acquired immunodeficiency syndrome

4. Correct: Cancer has a high incidence in the immune deficiency client and in the older adult with both of these risk factors together, this one is the highest risk for cancer. 1. Incorrect: Although smoking is a known environmental carcinogen, this one risk factor alone is not the highest risk. 2. Incorrect: These are known environmental carcinogens, but do not rank as highly as aging and immune deficiency. 3. Incorrect: Birth defects are not a risk factor for cancer.

The parents of a child hospitalized with cystic fibrosis have been given discharge instructions. The nurse knows that teaching has been successful when the parents make what statement? 1. "Our child will need to have a gluten free diet." 2. "The enzymes should be given at bedtime daily." 3. "Salt needs to be decreased in our child's diet." 4. "We need to prepare high calorie, high fat meals."

4. Correct: Cystic fibrosis is an inherited disorder in which abnormally viscous secretions affect the respiratory and digestive systems. Because the client is unable to absorb nutrients, several dietary adaptations are crucial, including frequent small meals along with digestive enzymes to help the client process food. The meals should be high calorie, high fat with increased amounts of sodium to help stabilize fluids. 1. Incorrect: A gluten free diet is not associated with cystic fibrosis. This special diet is generally required for clients with Celiac disease and certain food allergies, although clients with either of these diseases will need the addition of fat soluble vitamins A, D, E and K. This statement by the parents indicates the need for further teaching. 2. Incorrect: Pancreatic digestive enzymes, such as Creon or Pancreaze, must be given with every meal or snack in order to help the digestive system absorb nutrients properly. Because clients with cystic fibrosis need frequent small meals throughout the day, digestive enzymes must also be provided throughout the day with any food. 3. Incorrect: Clients with cystic fibrosis lose abnormally large amounts of salt in sweat, and the glands are unable to reabsorb needed sodium into the body system. Rapid dehydration is common due to decreased sodium levels, which are exacerbated during exercise or hot weather. These clients are encouraged to increase salt intake.

What task by the RN should be performed first? 1. Changing a burn dressing that is scheduled every four hours. 2. Administering scheduled IV antibiotic. 3. Teaching a new diagnosed diabetic about diet and exercise. 4. Assessing a newly admitted client.

4. Correct: The admit assessment should be done first. It is important to initiate the assessment and physical exam within one hour of being admitted to the unit or floor. The assessment and plan of care should be completed within 8 hours of admission. 1. Incorrect: The other clients' needs are important, but are scheduled and established in a routine. These routines can be continued once the new client's assessment has been completed. 2. Incorrect: This is not a priority based on the information in the question. The scheduled IV antibiotic administration can be administered within the appropriate time frame. 3. Incorrect: A newly diagnosed diabetic is not always ready for teaching, so this is not priority. The nurse should identify when the client is ready to learn. This teaching session can occur prior to or after assessing the new client.

The nurse approaches a client who entered the emergency department following a fall down a flight of stairs. The client is unresponsive with snoring and wheezes with respirations. How would the nurse best open the client's airway? 1. Endotracheal tube (ET) 2. Head tilt-chin lift maneuver 3. Oropharyngeal airway 4. Jaw thrust maneuver

4. Correct: This is a trauma client that could possibly have a C-spine injury. The jaw thrust maneuver will open the client's airway without manipulating the client's C-spine. 1. Incorrect: The endotracheal (ET) tube is a device for maintaining an open airway, not for opening it. 2. Incorrect: This is a trauma client who may have a C-spine injury. The head tilt-chin lift maneuver would manipulate the client's C-spine therefore is not used with this client to open the client's airway. 3. Incorrect: The oral airway is a device for maintaining an open airway, not for opening it.

In which phase of postpartal psychological adaption would discharge teaching regarding infant care most likely be successful? taking in letting go taking hold resolution

Beginning after completion of the taking-in phase, the taking-hold phase follows taking in lasts about 4 to 5 weeks. At this time, the client is most ready is ready to learn self-care and infant care skills. In the taking-in phase, the client focuses more on sleep. The letting-go phase is the final phase of postpartal psychological adaptation. This phase is characterized by readjustment, viewing the infant as a separate being, refocusing on her relationship with her partner, and maternal role attainment. Resolution is not considered an accepted phase of postpartal psychological adjustment.

A nurse is providing in-home management instructions to the parents of a child who is receiving desmopressin acetate (DDAVP). What is the most important instruction the nurse to include? Give DDAVP only when urine output begins to decrease Cleanse skin with alcohol before application of the DDAVP dermal patch Increase the DDAVP dose if polyuria occurs just before the next scheduled dose Call the healthcare provider if the child has an upper respiratory infection or allergic rhinitis

Excessive nasal mucus, associated with upper respiratory infection or allergic rhinitis, may interfere with DDAVP absorption because it is given intranasally. Parents should be instructed to contact the health care provider for advice in altering the hormone dose during times when nasal mucus may be increased. The DDAVP dose should remain unchanged, even if the child is experiencing polyuria just before the next dose to avoid over medicating the child.

A primiparous client who underwent a cesarean birth 30 minutes ago is to receive Rho(D) immune globulin. The nurse should administer the medication within which time frame after birth? 12 hours 24 hours 72 hours 48 hours

For maximum effectiveness, Rho(D) immune globulin should be administered within 72 hours postpartum. Most Rh-negative clients also receive Rho(D) immune globulin during the prenatal period at 28 weeks' gestation and then again after birth. The drug is given to Rh-negative mothers who have a negative Coombs test and give birth to Rh-positive neonates. If there is doubt about the fetus's blood type after pregnancy is terminated, the mother should receive the medication

The nurse is teaching the parents of a child with impetigo about care. Which statement by the parents indicate further teaching is needed? 1. "We will not allow bathing until the scabs are healed." 2. "The skin and crusts will be washed daily with soap and water." 3. "Lotions should not be applied to the lesions, so they remain dry." 4. "We will apply the antibiotic ointment to the lesions after removing the crusts."

Impetigo is a superficial infection of the skin caused by staphylococci, streptococci, or multiple bacteria. It is contagious and may spread to other parts of the skin or to other members of the family who touch the patient's or use towels or combs that are soiled with exudate of the lesions. Impetigo is particularly common in children living in poor hygienic conditions. The lesions are most commonly seen on the face or extremities. They begin as small, red macules which quickly become discrete, thin-walled vesicles that rupture and become covered with a loosely adherent honey-yellow crust. The crusts are easily removed to reveal smooth, red, moist surfaces on which new crusts soon develop. Option 1 is correct. The parents need further teaching. Lesions must be soaked or washed to remove the central site of bacterial growth. Bathing is needed. Option 2 is incorrect. The parents understand teaching with this statement. Washing the crusts daily with soap and water and not allowing the skin to dry promote quick healing of lesions. Option 3 is incorrect. The parents understand teaching with this statement. The lesions should have antibiotic ointment applied several times daily for 5 to 7 days. Option 4 is incorrect. The parents understand teaching with this statement. Lesions should first be soaked or washed with soap solution to remove the central site of bacterial growth, giving the topical antibiotic an opportunity to reach the infected site. After the crusts are removed, the prescribed topical antibiotic cream is applied.

A client hospitalized with a deep vein thrombosis (DVT) is on a heparin infusion. The client asks the nurse why it is necessary to have blood drawn every six hours. What is the best explanation for the nurse to provide to the client? 1. "The medicine might make your blood much too thin." 2. "It helps us monitor and adjust the dose to work better." 3. "It is required for anyone getting heparin intravenously." 4. "The test results tell us whether the treatment is working."

From the scenario, you know the client is on a heparin infusion and a venipuncture is done every 6 hours. As the nurse, you know blood is drawn for an activated partial prothrombin time (PTT), which is standard protocol for heparin. Heparin is not a thrombolytic agent in that it does not cause lysis of a clot, though it prevents a clot from getting bigger. Heparin has a short half-life, and must be monitored in order to maintain a consistent blood level. The primary healthcare provider provides a sliding scale to which the nurse will compare the lab results. The infusion rate is then adjusted based on whether the PTT is too low, or too high. Remember that if the client is discharged home on warfarin, that medication must be started several days before the heparin is discontinued to establish a safe blood level. Option 1: Not a good response. The client is frustrated about such frequent blood draws and therefore the explanation should be informative. This response by the nurse may frighten the client and does not actually explain the purpose of every 6 hour blood draws. Option 2: Wonderful! This answer directly clarifies the purpose and rationale for constant blood work without frightening the client or being overly technical. If the client needs more details, this open approach provides the opportunity for further discussion. Option 3: Definitely not. The nurse has not provided the client with a clear rationale of the blood work every 6 hours. This would be frustrating and not informative. Option 4: Not quite. Such a response is not really focused on why the client needs blood drawn every 6 hours, which is what the question was! The nurse could respond more effectively without becoming overly technical

The mother of an 8-year-old child with a fluid restriction of 1,000 mL/day is staying in the child's room. Which intervention would be most appropriate for the nurse to include in the child's plan of care? Discuss the fluid restriction with the mother and child, and allow them to decide how to allocate the fluids over the 24 hours. Explain to the mother that hospital personnel will assume the responsibility for providing fluids to the child. Let the child eat jello if fluid limits have been met. Tell the mother exactly how much fluid the child can have each hour, and show her examples of the amount.

Planning the child's fluid restriction with the mother and child is most appropriate because the mother and child would best know the child's usual pattern of fluid intake. Doing so also provides the mother with a feeling of some control over her child's situation and helps to promote compliance.

The nurse is caring for a client with severe diarrhea. The nurse recognizes that the client is at risk for developing which of the following acid-base imbalances? Metabolic acidosis Respiratory acidosis Metabolic alkalosis Respiratory alkalosis

The client is at risk for developing metabolic acidosis. Metabolic acidosis is caused by diarrhea, lower intestinal fistulas, ureterostomies, and use of diuretics; early renal insufficiency; excessive administration of chloride; and the administration of parenteral nutrition without bicarbonate or bicarbonate-producing solutes (e.g., lactate)

An outpatient client who has been receiving haloperidol for two days develops muscular rigidity, altered consciousness, a temperature of 103° F (39.4° C), and trouble breathing on day 3. The nurse interprets these findings as indicating which complication? neuroleptic malignant syndrome tardive dyskinesia extrapyramidal adverse effects drug-induced parkinsonism

The client is exhibiting hallmark signs and symptoms of life-threatening neuroleptic malignant syndrome induced by the haloperidol. Tardive dyskinesia usually occurs later in treatment, typically months to years later. Extrapyramidal adverse effects (dystonia, akathisia) and drug-induced parkinsonism, although common, are not life threatening.

When communicating with a client who has aphasia, which approaches are helpful? Select all that apply. Present one thought at a time. Avoid writing messages. Speak with normal volume. Make use of gestures. Encourage pointing to the needed object.

The goal of communicating with a client with aphasia is to minimize frustration and exhaustion. The nurse should encourage the client to write messages or use alternative forms of communication to avoid frustration. Presenting one thought at a time decreases stimuli that may distract the client, as does speaking in a normal volume and tone. The nurse should ask the client to point to objects and encourage the use of gestures to assist in communicating.

Propylthiouracil (PTU) is prescribed for a client with Graves' disease. Which symptom should the nurse teach the client to report? sore throat excessive menstruation constipation increased urine output

The most serious adverse effects of PTU are leukopenia and agranulocytosis, which usually occur within the first 3 months of treatment. The client should be taught to promptly report to the health care provider (HCP) signs and symptoms of infection, such as a sore throat and fever. Clients having a sore throat and fever should have an immediate white blood cell count and differential performed, and the drug must be withheld until the results are obtained. Painful menstruation, constipation, and increased urine output are not associated with PTU therapy.

A physician orders a palliative care consult for a client with end-stage chronic obstructive pulmonary disease who wishes no further medical intervention. Which step should the nurse anticipate based on her knowledge of palliative care? Decreasing administration of pain medications Reducing oxygen requirements Increasing the need for antianxiety agents Decreasing the use of bronchodilators

The nurse should anticipate that the physician will increase antianxiety agents during treatment to maintain comfort throughout the dying process. Bronchodilators, pain medications, and home oxygen therapy help promote client comfort. Therefore, they should be continued as part of palliative care.

The nurse is caring for a 3-month-old infant, who had a cleft palate and cleft lip surgical repair. Which assessment data would indicate a postoperative complication from the surgery? intermittent crying Logan bar in place swollen suture line suture line surrounded by erythema

There is a risk for infection in the suture line if it is not kept clean and dry. Signs of infection would include erythema or foul drainage from the suture line and fever. Crying intermittently is a normal assessment finding and the nurse should be prepared with liquids or formula. A suture line may be swollen in the immediate postoperative period, but its appearance will improve with time. A Logan bar may be used to hold the suture line in place.

A client had a mastectomy followed by chemotherapy 6 months ago. She reports that she is now "unable to concentrate at her card game" and "it seems harder and harder to finish my errands because I am so tired." What should the nurse suggest that the client do to manage the exhaustion? Take frequent naps. Limit activities. Increase fluid intake. Avoid contact with others.

This client is likely experiencing fatigue and should increase her periods of rest. The fatigue may be caused by anemia from depletion of red blood cells due to the chemotherapy. Asking the client to limit her activities may cause the client to become withdrawn. The information given does not support limiting activity. Increasing fluid intake will not reduce the fatigue. The information does not indicate that the client is immunosuppressed and should avoid contact with others.

A nurse is preparing a lecture about suicide. Which target audience would be most appropriate? 1. High school teachers 2. Girl Scout leaders 3. Support group of divorced parents 4. Hispanic immigrant farm workers

1. Correct: Among those who commit suicide, young men between the ages of 15-24 are more likely to commit suicide than young girls and women. The best line of defense is to teach about the warning signs of suicide to high school teachers, students, and parents of teens and young adults, particularly male. 2. Incorrect: Although teenage girls may attempt suicide, they are less likely than males to use a lethal method. Additionally, participation in groups such as the scouts will provide support for girls. 3. Incorrect: Joining a support group will help eliminate stress of being a single parent. Young males are more likely to attempt suicide by lethal means. 4. Incorrect: Hispanics have a lower suicide rate than Caucasians.

A client who was hospitalized with a diagnosis of schizophrenia tells the nurse, "My veins have turned to stone and my heart is solid!" How would the nurse identify this statement? 1. Depersonalization 2. Echopraxia 3. Neologism 4. Concrete thinking

1. Correct: Depersonalization, which is the unstable self-identity of an individual with schizophrenia may lead to feelings of unreality (the feeling that one's parts have changed or a sense of seeing oneself from a distance). 2. Incorrect: The client who exhibits echopraxia may purposelessly imitate movements made by others. 3. Incorrect: Neologism is the invention of new words by a psychotic client. 4. Incorrect: Concrete thinking, or literal interpretations of the environment, represents a regression to an earlier level of cognitive development.

Which symptoms would the nurse be likely to observe in the client who overdosed on diazepam? 1. Bradypnea 2. Bradycardia 3. Hyperthermia 4. Somnolence 5. Hyperreflexia 6. Psychosis

1., 2., & 4. Correct: Benzodiazepines are central nervous system (CNS) depressants. Diazepam is a benzodiazepine. They will slow respirations (bradypnea) and the heart rate (bradycardia). Somnolence (extreme, prolonged drowsiness) would be seen. 3. Incorrect: Benzodiazepines would not cause hyperthermia. 5. Incorrect: Benzodiazepines would diminish reflexes since it is a CNS depressant. 6. Incorrect: Psychosis is not a common symptom with CNS depression.

Which signs and symptoms, if noted by the nurse, would indicate that the client with hyperthyroidism is experiencing thyroid crisis? 1. Hyperkinesis 2. Bradycardia 3. Hypertension 4. Restlessness 5. Confusion

1., 3., 4., & 5. Correct: These are symptoms of thyroid crisis and should be reported immediately. 2. Incorrect: Tachycardia would occur, not bradycardia. Bradycardia is a symptom of hypothyroidism.

The nurse cares for a client who is scheduled for an upper GI series. The nurse teaches the client about the test. Which statement by the client indicates an understanding of the nurse's teaching? 1. I'll have to take a strong laxative the morning of the test. 2. I'll have to drink contrast while x-rays are taken. 3. I'll have a CT scan after I'm injected with a radiopaque contrast dye. 4. I'll have an instrument passed through my mouth to my stomach.

2. Correct: In an upper GI series (sometimes called a barium swallow test), the client swallows barium contrast while x-rays are taken. 1. Incorrect: Laxatives are taken the night before a colonoscopy to ensure stool is cleared from the colon. Waiting to take the laxative the morning of the test would be ineffective and uncomfortable for the client. 3. Incorrect: Radiopaque dye injected before a CT (computed tomography) scan is not part of a GI series. This would be a totally different diagnostic test from the upper GI. 4. Incorrect: In a gastroscopy (sometimes called a gastric endoscopy), a scope is passed through the mouth to the stomach to visualize the inner lining of the upper GI tract.

A nurse is caring for a 65 year-old client diagnosed with dehydration. The client has been receiving intravenous normal saline at 150 mL/hour for the past 4 hours. Which finding would the nurse need to notify the primary healthcare provider? 1. Blood pressure 136/84 2. Report of nausea 3. Anxiety 4. Urinary output at 50 mL/hour

3. Correct: Anxiety, restlessness, or a sense of apprehension is often the first sign/symptoms of acute pulmonary edema. 1. Incorrect: Blood pressure is normal. The number one concern right now is the anxiety: an early sign of pulmonary edema. 2. Incorrect: Although we would want to help the client having nausea, the anxiety is of upmost importance, as it might indicate acute pulmonary edema. 4. Incorrect: The client is dehydrated. A urinary output of 50 mL/hr, although low, is not at a critical level. Signs of pulmonary edema will take priority.

The nurse is working with the parents of a preschooler to help promote healthy sleep patterns of approximately 8-12 hours per night. Which intervention should assist the parents to achieve adequate sleep for their preschooler? 1. Offer a time of exercise prior to bedtime. 2. Follow a bedtime routine at least three or four nights per week. 3. Spend about 30 minutes with the preschooler prior to bedtime for stories, prayers, etc. 4. Do not encourage your preschooler to take a toy to bed.

3. Correct: Rituals help the preschooler to feel secure. Quiet time to read, tell stories, and say prayers prepares the child for sleep. 1. Incorrect: Stimulation of activity before bedtime impedes sleep. 2. Incorrect: The routine should be maintained each night if at all possible. Only through routine does the child feel secure in preparation for bedtime. 4. Incorrect: A special toy helps the child to feel secure and adds to the nighttime routine.

What impaired functions does the nurse expect to observe in the client admitted with an injury to the frontal lobe of the brain? 1. Decreased sensation to touch. 2. Impaired vision. 3. Impaired speech. 4. Decreased concentration. 5. Decreased hearing.

3., & 4. Correct: The frontal lobe is responsible for motor control, ability to speak words, concentration, memory, and judgment. 1. Incorrect: This is the function of the parietal lobe. 2. Incorrect: This is the function of the occipital lobe. 5. Incorrect: This is the function of the temporal lobe.

The nurse is caring for a client following spinal surgery. The client is placed on methylprednisolone. What additional drug therapy would the nurse expect to be prescribed with methylprednisolone? 1. Pantoprazole 2. Phenytoin 3. Imipramine HCI 4. Aminocaproic acid

1. Correct: A potential side effect of methylprednisolone is a peptic ulcer. The primary healthcare provider will prescribe a proton pump inhibitor or H2 blocker to prevent this side effect. 2. Incorrect: Phenytoin is an anticonvulsant. Seizures are not a side effect of methylprednisolone. 3. Incorrect: Imipramine HCI is an antidepressant which is not routinely given with methylprednisolone (Although mood changes can occur with steroid administration, anti-depressants are not routinely given). 4. Incorrect: Aminocaproic acid is given when clients are bleeding. Bleeding is not a side effect of methylprednisolone.

A client is taking a nonsteroidal anti-inflammatory drug (NSAID) for the relief of joint pain. A gastrointestinal bleed is suspected. Which laboratory value alerts the nurse to the possibility that the client is chronically losing small amounts of blood? 1. Prolonged bleeding time 2. Elevated reticulocyte count 3. Decreased platelet count 4. Elevated bands

2. Correct: Elevated reticulocyte count indicates increased production of RBCs. If a client is chronically losing blood, the body's response is to increase RBC production, so the retic count would increase. 1. Incorrect: Prolonged bleeding times occur with liver problems. 3. Incorrect: A decreased platelet count will cause bleeding but will not tell the nurse if there is chronic bleeding. 4. Incorrect: Elevated bands are a part of the WBC differential and are increased with acute infection.

What should the nurse teach the client following a right knee arthroscopy? 1. Apply ice to right knee continuously for the first 24 hours. 2. Elevate the right knee when sitting. 3. Notify the primary healthcare provider of tingling in the right leg. 4. Gradually start an exercise program to prevent scarring. 5. Place a plastic bag over wound when showering.

2., 3., 4. & 5. Correct: Elevating the joint for several days will reduce swelling and pain. Tingling to the extremity could mean nerves have been damaged. Exercise is gradually started to strengthen muscles surrounding the joint and prevent scarring of surrounding soft tissues. The client needs to keep the site as clean and dry as possible. 1. Incorrect: Continuous ice can cause tissue damage.

A client of Jewish faith has requested a Kosher diet. Which food tray would the nurse provide to the client? 1. Medium rare steak, potato salad, peas and coffee 2. Ham sandwich, chips, fruit salad and juice 3. Broiled white fish, baked potato, mixed salad and tea 4. Baked chicken, vegetable medley, rice and milk

3. Correct: Fish is allowed if it has fins and scales. Shellfish is not kosher. Pasta, potatoes, salads and tea are allowed. 1. Incorrect: Although steak is allowed, all traces of blood must be gone. 2. Incorrect: No pork products are allowed, so no bacon, ham, or sausage. 4. Incorrect: Milk is not allowed at the same time as meat. There should be at least three hours separating the two.

The client has just returned from electroconvulsive therapy (ECT) and is very drowsy. What is the position of choice for the nurse to place the client in until full consciousness is regained? 1. Supine 2. Fowler's 3. Lateral 4. High Fowler's

3. Correct: When someone is very sedated and not fully conscious, we want them on their side so the airway remains open and the secretions can drain. 1. Incorrect: No, the jaw will fall back, the tongue will block the airway, and the client will have airway obstruction, either partial or maybe even life-threatening. 2. Incorrect: No, if you sit a client up who is not fully conscious, the client's head tips forward and blocks the airway. 4. Incorrect: Again, head may fall forward and block airway.

The nurse is caring for a client receiving total parenteral nutrition (TPN). Which assessment would require the nurse to intervene? 1. TPN has been hanging for 12 hours 2. Central venous catheter's dressing is clean and dry 3. TPN fluid is room temperature when beginning administration 4. TPN appears oily in consistency

4. Correct: Do not use TPN if it looks curdled, oily, or has particles in it. This is an indication that something is wrong with the solution and could harm the client if given. 1. Incorrect: This TPN does not need to be replaced at 12 hours. It can infuse for 24 hours. 2. Incorrect: This is a description of an occlusive clean dressing at the insertion site. This description would not require intervention. 3. Incorrect: TPN should be at room temperature when beginning administration. Solutions that is too cold could cause vasoconstriction and undue harm to the client.

The homecare nurse is visiting a client to assess the response to new medications ordered for benign prostatic hyperplasia (BPH). What symptoms reported by the client would indicate to the nurse the medications are not working? 1. Bladder pain 2. Fever with chills 3. Urinary frequency 4. Terminal dribbling 5. Nighttime sweats

1, 3 and 4. CORRECT: Symptoms of benign prostatic hyperplasia are very similar to those of a urinary tract infection. As the prostate enlarges and presses against the bladder wall, it becomes more difficult for a client to start and maintain a stream of urine, or even to completely empty the bladder. Medications prescribed for this disorder are meant to shrink the prostate, allowing urine to flow easily when voiding. When the medications are ineffective, the client again experiences the original symptoms such as bladder pain, urinary frequency and a tendency to continue 'dribbling' urine after the bladder is emptied. The client may then need a different medication or a change in the dose currently prescribed. 2. INCORRECT: The symptoms of fever with chills are related to infection rather than benign prostatic hyperplasia. Although untreated BPH may lead to a urinary infection because of retained urine, these two symptoms do not relate directly to this prostate disorder. 5. INCORRECT: Nighttime sweats are not associated with benign prostatic hyperplasia. Nighttime sweats can be associated with tuberculosis.

A client with Hepatitis C has returned from surgery with a total laryngectomy. The nurse knows that what personal protective equipment is necessary when providing trach care? (Select All That Apply). 1. Face mask 2. Shoe covers 3. N-95 mask 4. Goggles 5. Gloves 6. Gown

1, 4, 5 and 6. CORRECT: The client has had a total laryngectomy which will initially produce large amounts of thick, bloody mucus. Hepatitis C is transmitted through blood and body fluids. During trach care, the nurse needs to be protected by specific personal protective equipment (PPE's). For this procedure, the nurse should utilize gown, gloves, goggles and face mask. 2. INCORRECT: Tracheostomy care is completed in close proximity to the client. Splattering of blood and body fluids on the floor is unlikely, so shoe covers are unnecessary. 3. INCORRECT: The N-95 face mask is a specially fitted mask used by nurses when providing care for clients with active tuberculosis. It is not necessary for a client with Hepatitis C.

The nurse receives new healthcare provider prescriptions on a client diagnosed with Addison's disease. Which prescriptions should the nurse recognize as being inappropriately written and requiring clarification from the prescriber? 1. Weigh QD 2. IV of normal saline at 125 mL/hr 3. MRI of pituitary gland 4. Fludrocortisone acetate 0.1 mg by mouth T.I.W. 5. Dehydroepiandrosterone DHEA sulfate 5 mg by mouth every other day

1. & 4. Correct: Use "daily" or "every day". QD is an unapproved abbreviation. T.I.W. stands for three times a week; however, it is an unapproved abbreviation. Use "three times a week". 2. Incorrect: This is a correct action and is written properly. 3. Incorrect: The primary healthcare provider may suggest an MRI scan of the pituitary gland if testing indicates the client might have secondary adrenal insufficiency. This is an approved abbreviation. 5. Incorrect: This is written correctly and may be given to women to treat androgen deficiency.

A client arrives by ambulance after being thrown from a horse. The client is pale, clammy and tachycardic with bruising over left upper abdominal quadrant. The nurse is aware what prescription by the primary healthcare provider takes priority? 1. Obtain blood for type and cross match. 2. Administer hydromorphone IV for pain. 3. Increase Lactated Ringers to 150 mL/hour. 4. Send client to radiology for stat CAT scan.

1. CORRECT: The signs and symptoms displayed by the client suggest a ruptured spleen and shock. The greatest concern in this situation is internal bleeding and possible emergency surgery. The client will need blood; therefore, the nurse should immediately obtain blood for type and cross match. 2. INCORRECT: There is no indication in the scenario the client has pain. Pain medication should never be administered while the client is still being assessed or is in shock. 3. INCORRECT: Fluids are crucial for clients in shock and increasing the Lactated Ringers to 150 mL/hr. is important to help maintain blood pressure. However, this is not the nurse's priority action. 4. INCORRECT: A CAT scan is often prescribed prior to surgery to verify the extent of splenic injury and the amount of blood in the abdominal cavity. Though the order is written as 'stat', this is not the nurse's priority. Transporting an unstable client to another department requires preparation.

The nurse is caring for a burn client in the emergent phase. The client becomes extremely restless while on a ventilator. What is the priority nursing assessment? 1. Patency of endotracheal tube. 2. Adventitious breath sounds. 3. Fluid in the ventilator tubing. 4. Ventilator settings.

1. Correct: With restlessness, think hypoxia so the nurse should start assessment with airway first. Check for patency of the ET tube. If this is patent, then the other options would be next. 2. Incorrect: This is the next best answer, but hypoxia and airway comes first. 3. Incorrect: This is the third step. Rule out the other two before checking tubing for kinks or obstructions. 4. Incorrect: Start with the client first. Then move toward the ventilator. Always assess the client first.

The nurse suspects a client admitted with myasthenia gravis is going into a cholinergic crisis. Which signs and symptoms would validate the nurse's suspicions? 1. Abdominal cramping 2. Lethargy 3. Salivation 4. Hypertension 5. Lacrimation 6. Miosis

1., 2., 3., 5., & 6. Correct: The signs of cholinergic crisis include Diarrhea and abdominal cramping, Urination increased, Miosis (pinpoint pupils), Bradycardia, Emesis (nausea and vomiting), Lacrimation, Lethargy, Salivation. Remember this: DUMBELLS as a mnemonic to help you recall these signs and symptoms. 4. Incorrect: Hypertension is not a sign of cholinergic crisis. Muscles get weaker so BP would go down.

What risk factors should the nurse include when conducting a class about type 2 diabetes mellitus? 1. Fat distribution greater in abdomen than in hips. 2. Being underweight. 3. Having type 1 diabetes as a child increases risk for type 2 diabetes. 4. Caucasians are more likely to develop type 2 diabetes than Hispanics. 5. Polycystic ovary syndrome.

1., & 5. Correct: If the body stores fat primarily in the abdomen, risk of type 2 diabetes is greater than if body stores fat elsewhere, such as hips and thighs. Women with polycystic ovary syndrome have increased risk of diabetes. 2. Incorrect: Being overweight is a primary risk factor for type 2 diabetes. The more fatty tissue, the more resistant cells become to insulin. 3. Incorrect: A type 1 diabetic will remain a type 1 diabetic. 4. Incorrect: Blacks, Hispanics, American Indians, and Asian Americans are more likely to develop type 2 diabetes than Caucasians are.

A nurse who has never had varicella has been exposed to a client diagnosed with herpes zoster. What actions should the nurse take? 1. Notify the infection control nurse. 2. Continue to care for client as varicella and herpes zoster are not related. 3. Go to the lab to have a Tzanck smear performed. 4. Obtain herpes zoster vaccine for protection from this exposure. 5. Receive the varicella-zoster immune globulin within 96 hours of exposure.

1., & 5. Correct: Notify the person responsible for infection control to get post-exposure treatment initiated within a timely manner. For persons who are susceptible, the varicella-zoster immune globulin should be given within 96 hours of exposure. The infection of herpes zoster is contagious until the crusts have dried and fallen off the skin. 2. Incorrect: Varicella is chickenpox and herpes zoster is shingles. Both are closely related. Exposure to herpes zoster by someone who has not had varicella places the person at risk for developing herpes zoster. 3. Incorrect: A Tzanck test consists of examining tissue from the lower surface of a lesion in a vesicular condition to determine cell type. The Tzanck test is not associated with immunity from the varicella-zoster virus. 4. Incorrect: The vaccine will not prevent the nurse from developing shingles from this exposure. The nurse needs immune globulin for immediate protection.

The nurse determines that a client does not have an advance directive. The daughter is designated to make healthcare decisions in the event that the client becomes incapacitated or unable to make informed decisions. Which nursing actions are appropriate for this client? 1. Document the client's statement in the client's own words. 2. Provide information on advance directives to the client. 3. Inform the client that personnel are available to assist with completing an advance directive. 4. Avoid inquiring about a client's advance directive as this could cause the client anxiety and concern. 5. Ask the daughter if she agrees with her mother's decision.

1., 2. & 3. Correct: The nurse should document the client's statement in the client's own words. The nurse should provide the client with information on advance directives and assurance that there are hospital personnel to assist with completing the advance directive. 4. Incorrect: The nurse who avoids inquiry about a client's advance directive is not serving the client's best interests. The nurse should explain to the client that the law requires all clients be asked about the existence of an advance directive at the time of hospital admission. Preparing an advance directive ensures that the client's wishes will be followed in the event that the client is unable to make healthcare decisions. 5. Incorrect: Providing information is the appropriate nursing action, not questioning the daughter.

Which interventions decrease risk of infection or damage to delicate tissue when the nurse is changing a wound dressing? 1. Warm cleansing solutions to body temperature. 2. Clean the wound when there is drainage present. 3. Use cotton balls to clean the suture site. 4. Use sterile gauze squares to dry the wound 5. Use sterile forceps when cleaning the wound.

1., 2. & 5. Correct: Using cleansing solutions at body temperature enhances the healing process by not lowering the temperature of the wound and enhancing circulation to the wound bed. Drainage should be removed so that it does not become infected because drainage and exudate can create an environment where bacteria can thrive. Sterile forceps should be used so that contaminated hands/gloves do not increase the risk of infection at the wound site. 3. Incorrect: Cotton balls may leave small cotton filaments behind that may serve as a site for infection. 4. Incorrect: Moisture is important for the healing process, so drying the wound could delay the healing process and cause undue harm to the client. Cells that are kept moist and hydrated promote epidermal growth. This will promote the healing of the cell base of the wound.

What should a community health nurse include when planning a presentation on prevention and early detection of colon cancer? 1. Maintain a diet high in fruits, vegetables, and whole grains. 2. Exercise regularly. 3. Regular screening should begin at age 30. 4. Yearly guaiac-based fecal occult blood test. 5. Flexible esophagogastroduodenoscopy every 5 years.

1., 2., & 4. Correct: A diet high in vegetables, fruits, and whole grains has been linked with a decreased risk of colorectal cancer; whereas, a diet high in red meats, processed meats, and cooking meats at very high temperature (frying, broiling or grilling) creates chemicals that may increase the risk for colorectal cancer. There is a greater risk of developing colorectal cancer in individuals who live a sedentary life style. The guaiac-based fecal occult blood test detects blood in the stool through a chemical reaction. This test is done yearly. 3. Incorrect: If there are no identified risk factors (other than age), regular screening should begin at age 50. 5. Incorrect: Flexible sigmoidoscopy looks at the rectum and colon to detect polyps and colon cancer. For people who have none of the risks described earlier, digital rectal examination and testing of the stool for hidden blood are recommended annually beginning at age 40. Flexible sigmoidoscopy is recommended every 5 years at age 50 or older. A double contrast barium enema every 5 to 10 years and colonoscopy every 10 years are acceptable alternatives.

What discharge instructions should the nurse provide to the client post abdominal hysterectomy? 1. Ambulate at least 3-4 times per day. 2. Notify the primary healthcare provider if there is a yellow discharge from the surgical wound. 3. Swimming is allowed if staples were used to close the skin. 4. Press a pillow over incision when coughing to ease discomfort. 5. Apply moist heat to surgical site the first couple of days for pain relief.

1., 2., & 4. Correct: The client should get up and move to prevent complications such as deep vein thrombosis (DVT), pneumonia, constipation, etc. The healthcare provider should be notified if the surgical wound is bleeding, red and warm to touch or has a thick, yellow, or green drainage. Pressing a pillow over incision when coughing or sneezing will ease discomfort and protect the incision. 3. Incorrect: Do not go swimming or soak in a bathtub or hot tub until the primary healthcare provider says it is ok. You worry about infection. 5. Incorrect: In the first couple of days, an ice pack may help relieve some pain at the site of surgery. Remember NCLEX wants you to think safety first when it comes to the use of heat.

The home health nurse is assessing the home environment for possible irritants that could increase/precipitate symptoms of respiratory problems. Which assessment questions would be important to determine level of risk? 1. What type of heat do you use in the home? 2. Does anyone in the home have hobbies that involve sanding of wood or use of chemicals? 3. Is there anyone in the home who smokes? 4. Do you routinely use aerosol sprays for personal care or cleaning? 5. Is your water supply treated by a municipal agency?

1., 2., 3. & 4. Correct: Presence of wood smoke could increase respiratory problems. Poorly vented gas heaters could increase carbon monoxide in the environment. Use of solvents or other agents that produce irritating fumes could increase risk. The particles from the sanding could irritate the respiratory tract as well. Second-hand smoke is irritating to the respiratory tract. Aerosols could trigger respiratory problems. 5. Incorrect: Water safety would not necessarily increase respiratory risk.

Which prescriptions are appropriate for the nursery nurse to initiate on a newborn prior to discharge home? 1. Hepatitis B vaccine 2. Erythromycin Ointment 3. Vitamin K 4. Lanolin 5. PKU Screening

1., 2., 3. & 5. Correct: This vaccine is recommended at birth to decrease the incidence of hepatits B virus. Mandatory prophylactic agent is applied in newborn's eyes as precaution against ophthalmia neonatorium. Vitamin K (Aquamephyton) routine injection to prevent hemorrhagic disease of newborn. PKU-Screening for phenylketonuria is not reliable until the newborn has ingested an ample amount of the amino acid, phenylalanine, a constituent of both human and cow's milk. Nurse must document initial ingestion of milk and perform test at least 24 hours after that time. This test is thus done just prior to discharge. 4. Incorrect: Lanolin is not something that is applied on newborns when admitted to the nursery.​ Lanolin is an emollient for the skin. The normal newborn does not need an emollient applied to the skin.

A client has been admitted to the unit with acute pyelonephritis. What interventions should the nurse include in this client's plan of care? 1. Observe for changes in mental status. 2. Assist client to restroom. 3. Monitor temperature every 4 hours. 4. Help the client get in a comfortable position to void. 5. Instruct client to void every 30 minutes while ill.

1., 2., 3., & 4. Correct: Changes in mental status may signify septic shock. Help clients to the restroom, use bedpans/urinals, and get in a comfortable position to facilitate urination. Pyelonephritis is a bacterial infection and fever is a clinical manifestation the nurse needs to monitor. 5. Incorrect: Instruct to void every 2-3 hours. This will prevent the buildup of urine in the bladder. Every 30 minutes is excessive.

Which assessment findings would the nurse expect to see in a client diagnosed with idiopathic thrombocytopenic purpura (ITP)? 1. Ecchymosis 2. Bleeding gums 3. Palpable spleen 4. Pain 5. Petechiae

1., 2., 3., & 5. Correct: The word thrombocytopenia means low platelets. Any client with low platelets is at risk for bleeding, which is indicated by ecchymosis (bruising), bleeding gums, and petechiae (red to purple dots on the skin, 1-3 mm in size). Spleen and liver are often slightly palpable. 4. Incorrect: Pain is not associated with ITP unless there are other associated problems. However, the stem of the question gave no indication that other problems exist.

The homecare nurse is providing family teaching on safety issues for a client diagnosed with Parkinson's disease. What adaptations should the nurse instruct the family to initiate? 1. Install grab bars on tub walls. 2. Place nightlights in hallways. 3. Add bran and fiber to daily diet. 4. Remove scatter rugs or loose cords. 5. Keep bedroom dark, cool and quiet. 6. Put tennis balls on wheels of walker.

1., 2., 3., 4., & 5. Correct: Parkinson's disease causes deterioration of the basal ganglia, ultimately impacting motor control and function. As muscles become stiff and rigid, mobility slows, resulting in poor coordination and loss of balance. Safety is a chief concern in all ADLs, requiring modifications in activity, nutrition, and the client's environment. Because Parkinson's disease affects mobility, modification such as grab bars and night lights are essential. Clients develop constipation because of decreased peristalsis, so adding bran and fiber can address impending bowel issues. Scatter, or throw, rugs along with loose extension cords on the floor create a fall risk because the client is unable to pick up feet during ambulation. The shuffling gait that develops increases the risk for falls. These clients also have problems with insomnia along with poor REM sleep, leading to daytime drowsiness. Making the bedroom conducive to sleep may help alleviate symptoms for a period of time. A dark, cool room with no distractions is the most appropriate sleep environment. 6. Incorrect: The proper method of utilizing a walker is to step into the walker, pause and then move it forward before stepping again. Even though clients with Parkinson's disease have a shuffling gait and stooped posture, sliding a walker with tennis balls on the wheels presents a serious safety issue. The client would not have the ability to control the speed or hold the walker steady while stepping into it.

A case manager is evaluating a client diagnosed with hemiplegia due to a cerebral vascular accident who will need assistive devices upon discharge. Which devices should the case manager include for this client? 1. Dinner plate food guards 2. Transfer belt 3. Raised toilet seat 4. Long handled shoe horn 5. Wide grip eating utensils 6. Button closures on clothes

1., 2., 3., 4., & 5. Correct: The goal is to promote self-care by the client as much as possible. The case manager should evaluate the need for assistive devices to help with eating, bathing, dressing, and ambulating. The dinner plate food guard will prevent food from being pushed off the plate. The transfer belt will provide safety for the client to get into a chair or back in bed. A raised toilet seat makes it easier for the client to sit on the toilet without falling. The long-handled shoe horn allows the client to put on shoes without assistance. Wide grip utensils accommodate a weak grip. 6. Incorrect: It is hard for someone with hemiplegia to use buttons. Velcro fasteners are best.

What should the nurse document after a client has died? 1. Time of death 2. Who pronounced the death 3. Disposition of personal articles 4. Destination of body 5. Primary healthcare provider's prescriptions 6. Time body left facility

1., 2., 3., 4., & 6. Correct: All of these should be documented. Also document consideration of and preparation for organ donation; family notified and decisions made; location of identification tags. 5. Incorrect: The primary healthcare provider's prescriptions do not need to be documented after a client dies.

A client diagnosed with rheumatoid arthritis has been prescribed dexamethasone orally as part of initial treatment therapy. What side effects should the nurse teach the client are expected? 1. Fatigue 2. Insomnia 3. Hypoglycemia 4. Truncal obesity 5. Increased appetite 6. Low blood pressure

1., 2., 4. & 5. Correct: Dexamethasone (Decadron) is a corticosteroid used short term to treat severe inflammation occurring in rheumatoid arthritis (RA). Expected side effects are associated with the body's response to excessive steroids in the system. Even short term use of corticosteroids will produce fatigue, secondary to insomnia, truncal obesity accompanied by thin extremities, and an increased appetite resulting in weight gain. Despite the short and intermittent use of corticosteroids for this auto-immune disease, some side effects remain permanently. 3. Incorrect: Excessive steroids in the body cause blood glucose levels to increase, resulting in hyperglycemia. Clients taking corticosteroids will need regular finger stick glucose monitoring while taking these medications for rheumatoid arthritis. 6. Incorrect: The body's response to increased corticosteroids in the system is an elevated blood pressure, often accompanied by headaches or nausea. Clients taking steroids will need to have blood pressure checked frequently.

A nurse is caring for a client who reports fatigue, weight loss, afternoon fevers, night sweats, cough, and hemoptysis. What interventions should the nurse initiate? 1. Wear an N95 respirator when caring for client. 2. Restrict fluid intake to 500 mL per day. 3. Position client in semi-Fowler's position. 4. Place client in a negative pressure airflow room. 5. Do not allow visitors for 48 hours.

1., 3. & 4. Correct: The nurse should suspect that the client is suffering from tuberculosis. Early pulmonary TB is asymptomatic. When the bacterial load increases, nonspecific symptoms of fatigue, weight loss, afternoon fevers, and night sweats may set in. As disease advances, cough, sputum production, and hemoptysis may appear. This client has the classic symptoms of TB and should be placed on airborne precautions. N95 respirator ensures that the nurse does not inhale the TB organism. Placing in a semi-Fowler's position reduces the work of breathing. 2. Incorrect: Unless contraindicated, 3-4 liters of fluid is needed per day to liquefy secretions. 5. Incorrect: Visitors are allowed if standard and airborne precautions are followed.

The nurse notes that a client has impaired swallowing as a result of a cerebrovascular accident (CVA). Which interventions are appropriate for the nurse to include in the plan of care? 1. Sit the client up at a 90° angle during meals. 2. Assist the client to hyperextend the head when preparing to swallow. 3. Encourage the client to sit up for 30 minutes after eating. 4. Educate a family member on the Heimlich maneuver. 5. Start the client on a thin liquid diet.

1., 3. & 4. Correct: This is the optimal position for chewing and swallowing without aspirating. Sitting up 30 minutes after completing a meal will prevent regurgitation of food. In case of choking, family members should know how to perform emergency measures such as the Heimlich maneuver. 2. Incorrect: The client should position the head in forward flexion in preparation for swallowing, called the "chin tuck". Hyperextension would cause aspiration. 5. Incorrect: The client should be started on thick liquid or pureed diet. Thickened or pureed foods are easier to swallow than thin liquids and prevent aspiration.

What should the nurse include when providing education to a client receiving tetracycline? 1. Wear long sleeves when going outside. 2. Take tetracycline on a full stomach. 3. Wait at least two hours after taking tetracycline prior to taking iron supplements. 4. Tetracycline can decrease the effectiveness of birth control pills. 5. Do not take this medicine after the expiration date on the label has passed.

1., 3., 4., & 5. Correct: Avoid exposure to sunlight or artificial UV rays (sunlamps or tanning beds). Tetracycline can make your skin more sensitive to sunlight and sunburn may result. Use a sunscreen (minimum SPF 15) and wear protective clothing if you must be out in the sun. Take tetracycline on an empty stomach and do not take iron supplements, multivitamins, calcium supplements, antacids, or laxatives within 2 hours before or after taking tetracycline. These products can make this medicine less effective. Tetracycline can make birth control pills less effective. Use a second method of birth control while you are taking this medicine to keep from getting pregnant. Throw away any unused tetracycline when it expires or when it is no longer needed. Do not take this medicine after the expiration date on the label has passed. Expired tetracycline can cause a dangerous syndrome resulting in damage to the kidneys. 2. Incorrect: Take tetracycline on an empty stomach and do not take iron supplements, multivitamins, calcium supplements, antacids, or laxatives within 2 hours before or after taking tetracycline. These products can make this medicine less effective.

A client who needs to have a stool specimen for an occult blood test is instructed by the nurse to avoid which substances two hours prior to testing? 1. Liver 2. Tomato 3. Ibuprofen 4. Sardines 5. Ascorbic acid

1., 3., 4., 5. Correct: The following foods can cause a false-positive reading: red meats, liver, turnips, broccoli, cauliflower, melons, salmon, sardines, and horseradish. Medications altering the test include aspirin, ibuprofen, ascorbic acid, indomethacin, colchicines, corticosteroids, cancer chemotherapeutic agents, and anticoagulants. Ingestion of vitamin rich foods can cause a false negative result. 2. Incorrect: A tomato is not on the food list for false-positive reading and do not have to be avoided.

A child diagnosed with AIDS is scheduled for grade school immunizations. Which immunizations are safe for the nurse to administer to the child? 1. MMR (measles, mumps, rubella) 2. DTaP (diphtheria, tetanus, pertussis) 3. VAR (varicella) 4. HiB (haemophilus influenza) 5. OPV (oral polio virus)

2 & 4. Correct: Children with AIDS are immunocompromised because of the HIV virus. Vaccines are crucial to provide protection against common childhood diseases. However, only vaccines which contain synthetic or inactivated viral components are acceptable for children with active AIDS. Diphtheria, tetanus, pertussis is inactive and is provided in multiple doses, starting at 2 months of age, with a booster at age 6. Haemophilus influenza is critically important since this flu virus can lead to meningitis, pneumonia or epiglottitis. This vaccine is also administered in multiple injections over a period of months, starting at 2 months, and then yearly throughout life. 1. Incorrect: The combination vaccine of measles, mumps, and rubella contains a live virus. Although research is ongoing, the Center for Disease Control (CDC) suggests while children diagnosed HIV+ may receive the vaccine, those with active AIDS should not be administered this vaccine. 3. Incorrect: Varicella is a live vaccine administered to protect children from chickenpox and the potential for shingles later in life. Though the disease and its dormancy in the body can have serious long-term effects, the vaccine is considered inappropriate for children with AIDS. 5. Incorrect: Oral polio vaccine contains the live polio virus and could be deadly to those with an immunocompromised system. The correct form of polio vaccine for AIDS clients is called IPV, or inactivated polio vaccine, and is given by injection.

Following nasal surgery, the nurse suspects a client has developed diabetes insipidus. The nurse knows what laboratory results provide evidence of diabetes insipidus? 1. White blood cells of 9,500 mm3 (9.5 x 10^9/L) 2. Urine specific gravity of 1.004 3. Serum sodium level of 149 mEq/L (149 mmol/L) 4. Hemoglobin of 20 g/dL (200 g/L) 5. Glucose of 100 mg/dL (5.6 mmol/L)

2, 3 and 4. CORRECT: Diabetes insipidus results when the body is deficient in anti-diuretic hormone (ADH), resulting in a fluid volume deficit and shock. Blood becomes concentrated and urine dilute because of extreme loss of water. Specific gravity is very low, as evidenced by a lab result of 1.004. However, serum levels of sodium (149 mEq/L (149 mmol/L)) and hemoglobin (20 g/dL (200 g/L)) are high due to concentration. 1. INCORRECT: Normal white blood cell count is 5,000 to 10,000 mm3 (5-10 x 10^9/L). This WBC result is normal and does not require action by the nurse. 5. INCORRECT: Diabetes insipidus is not related to the disease diabetes mellitus. The blood glucose level in this question is normal.

A client had a coronaryartery bypass surgery (CABG) x 3 performed 24 hours ago. What assessment findings would make the nurse suspect cardiac tamponade? 1. Bradycardia with wet lungs 2. Increased central venous pressure 3. Distended bilateral neck veins 4. A widening pulse pressure 5. Decreasing blood pressure

2, 3, and 5. CORRECT: Cardiac tamponade occurs when blood or fluid enters the pericardial sac, causing compression of the heart chambers. Such pressure prevents blood from either entering or leaving the heart, thus decreasing cardiac output. Central venous pressure (CVP) increases because of the compression of the right atria, but because no fluid is exiting the heart, blood pressure drops. Since the returning blood cannot enter the heart, neck veins become distended, though lungs remain clear. 1. INCORRECT: Because little blood is moving within the heart chambers, no fluid would back up into the lungs. The client's lungs remain clear, even though the cardiac output decreases and neck veins are distended. 4. INCORRECT: Widening pulse pressure is noted with increased intracranial pressure, not cardiac tamponade. Pulse pressure in tamponade would narrow since the heart is being compressed.

A primary healthcare provider informs the nurse to prepare for an amniotomy on a client who's labor has not progressed. What should the nurse assess for prior to the primary healthcare provider performing this procedure? 1. Fetal attitude 2. Fetal engagement 3. Fetal lie 4. Fetal position

2. Correct: Fetal engagement is important prior to rupturing the membranes so that the umbilical cord cannot prolapse. Fetal engagement is when the fetus is at station 0 (level of mom's ischial spines). 1. Incorrect: Fetal attitude is where the extremities and chin of the fetus are in relation to the fetal body. 3. Incorrect: Fetal lie refers to the maternal spine in relation to the fetal spine. 4. Incorrect: Fetal position tells us the presenting part of the fetus to mom's pelvis.

A client comes to the clinic reporting palpitations, as well as nausea and vomiting while taking metronidazole. The nurse notes that the client is flushed and has a heart rate of 118 bpm. Based on this information, what is the most important question for the nurse to ask the client? 1. "Do you take metronidazole on an empty stomach?" 2. "Are you using any products that contain alcohol?" 3. "How long have you had these symptoms?" 4. "What other medications are you currently taking?"

2. Correct: Flushing, nausea and vomiting, palpitations, tachycardia, psychosis are signs of disulfiram-type reaction seen when using products containing alcohol (cologne, after shave lotion, or path splashes) or ingesting alcohol products while taking metronidazole. 1. Incorrect: Although it is preferable to take metronidazole on an empty stomach, this is not the most important question to ask at this time. 3. Incorrect: How long the client has had these symptoms is not as important as whether the client is using any alcohol containing products. 4. Incorrect: Although the nurse needs to know what other medications the client is taking, it is not as important as knowing if the client is using any alcohol containing products.

A client diagnosed with systemic lupus erythematosus (SLE) has been started on hydroxychloroquine sulfate to decrease joint pain and swelling. What statement by the client indicates to the nurse the medication teaching has been effective? 1. "I will be prone to infections while on this medication." 2. "I need to see my eye doctor at least once every year." 3. "I might develop a red rash on my nose and cheeks." 4. "I can stop this medicine after my symptoms are gone."

2. Correct: Hydroxychloroquine sulfate(Plaquenil) is in the category of DMARDs (disease modifying anti-rheumatic drug) and was originally developed to treat or prevent malaria. When taken once or twice daily, this medication reduces swelling and joint pain while also decreasing skin problems in Lupus clients. Though there are relatively few side effects, the most serious is retinal toxicity which requires treatment by an ophthalmologist. It is imperative for clients on this medication to have an eye examination every 6 to 12 months. 1. Incorrect: This medication is an antimalarial which has been shown to decrease pain from arthralgia in clients with SLE. Minimal side effects are generally limited to gastrointestinal disturbances such as nausea or diarrhea. This medication does not increase the client's risk of infection at all. 3. Incorrect: Clients with SLE frequently develop the classic red "butterfly rash" across the nose and cheeks which becomes worse when exposed to the sunlight. This symptom occurs because of the disease process and is not related to any medications the client may be taking. 4. Incorrect: There are several categories of medications used to treat SLE; however, none of them should be stopped suddenly. The disappearance of symptoms generally indicates the medication regime is working well, and the client should never suddenly discontinue any medicine unless instructed to do so. Abruptly stopping this drug increases the risk of an exacerbation of symptoms such as nephritis or vasculitis.

The oncoming nurse has just received report and is preparing to make initial rounds. Which postpartum client should the nurse see first? 1. A primipara 6 hours postpartum saturating one peripad every two hours 2. A multigravida 1 hour postpartum and reporting intense perineal pain 3. A primigravida 12 hours postpartum with the uterine fundus at the umbilicus 4. A multigravida 72 hours postpartum with a brownish pink lochia discharge.

2. Correct: Intense perineal pain is a symptom of a perineal hematoma which is a medical emergency. 1. Incorrect: Expected findings for the postpartum period are described here. This is a normal peripad saturation and does not indicate a problem. 3. Incorrect: Expected findings for the postpartum period are described here. This is the proper position of the fundus 12 hours postpartum. 4. Incorrect: Expected findings for the postpartum period are described here also. A client postpartal 72 hours should have a brownish pink lochia discharge.

A client is scheduled for plateletpheresis. When taking the client's history, which information is most significant? 1. Allergies to shellfish 2. Date last donated 3. Time of last oral intake 4. Blood type

2. Correct: Platelet donors can have plateletpheresis as often as every 14 days. 1. Incorrect: Allergies to shellfish have nothing to do with withdrawing platelets from the client. 3. Incorrect: Time of last oral intake has no bearing on whether or not a client can donate platelets. 4. Incorrect: Blood type has no bearing on whether or not a client can donate platelets.

A client with a diagnosis of endocarditis and a new peripherally inserted cential catheter (PICC) line has been discharged home to receive daily intravenous antibiotics for six more weeks. The home health nurse is making an assessment visit today. What instruction by the nurse is most important initially? 1. Take antibiotics before dental procedures. 2. Brush and floss teeth at least twice daily. 3. Report any flu like symptoms immediately. 4. Include rest periods throughout the day.

2. Correct: Poor dental hygiene is one of the chief causes of endocarditis in adults, leading to growth of vegetation on heart valves, emboli, strokes, or even death. Instructions on proper oral care is considered primary or preventative teaching and encourages the client to take an active role in personal health care. Decreasing mouth bacteria or disease will decrease the potential for a reoccurrence of endocarditis. 1. Incorrect: Although primary healthcare providers may order antibiotics prior to a dental visit, it depends on what procedure the dentist is going to perform. Invasive mouth procedures where bleeding is likely generally require pre-visit antibiotics. However, this is not the most important information by the nurse initially. 3. Incorrect: Flu like symptoms are an indication of a possible exacerbation or reoccurrence of endocarditis. The client would be instructed to report such signs as fever, chills, malaise, or night sweats immediately to the primary healthcare provider. While it is important for the client to understand what to report, preventative measures are more important at this time. 4. Incorrect: Infection within the heart is very serious and, despite aggressive treatment, may have lasting effects on the client's cardiovascular system. Decreasing the workload of the heart during treatment and recovery time would certainly assist with the healing process. However, the need for frequent rest periods throughout the day is determined by a variety of factors, such as the client's age and morbidity factors, general health, amount of damage to the heart, and response to antibiotics. Rest is not the most important instruction the nurse must present initially.

The six bed Labor and Delivery area is full when the Emergency Department nurse calls for a bed for a woman reporting low back pain, pelvic pressure and increased vaginal discharge at 36 weeks gestation. Which would be the most appropriate action for the charge nurse? 1. Transfer a G4P4 who delivered full-term twins one hour ago to the antepartum/postpartum floor. 2. Transfer a G3 P3 who delivered an 8 lb. newborn three hours ago to the antepartum/postpartum floor. 3. Transfer an 8 hour postpartum G1P1 on Magnesium Sulfate for eclampsia from the LDR unit to the ante/postpartum unit. 4. Request that the new client be admitted to the antepartum/postpartum floor.

2. Correct: The client and newborn are not in any present distress. Also the delivery occurred 3 hours ago. This client would not be a risk and could be cared for on the antepartum/postpartum floor. 1. Incorrect: This client is at high risk for hemorrhage due to still being in the fourth stage of labor and over distention of her uterus with a term multiple gestation. This client needs to stay in current location for close monitoring. 3. Incorrect: Close monitoring and frequent vital signs are required since central nervous system alterations and respiratory depression are common side effects of Magnesium Sulfate. 4. Incorrect: This client has subtle signs and symptoms of preterm labor and needs close monitoring. Ante and postpartum unit is for stable clients.

After a thoracotomy, which interventions will the nurse initiate to reduce the risk of acute respiratory distress? 1. Allow 4 hours of rest between deep breathing and coughing exercises. 2. Splint the incision during deep breathing and coughing exercises. 3. Have the client drink a glass of water before coughing. 4. Perform percussion and vibration every 2 hours. 5. Promote incentive spirometer use several times per hour while awake.

2., & 5. Correct: Splinting helps with the ability to control pain and produce an effective cough. Incentive spirometry encourages deep inspiratory efforts, which are more effective in re-expanding alveoli than forceful expiratory efforts. 1. Incorrect: They need to cough more often than every 4 hours. It is the best when this is done every 2 hours. 3. Incorrect: It takes longer than a few minutes to liquefy secretions and, if the stomach is full, vomiting may occur which would put the client at risk for aspiration. 4. Incorrect: After the surgery, we do not want to percuss and vibrate the incision. Besides being extremely painful, this could potentially disrupt the suture line.

Which task would be appropriate for the Labor, Delivery, Recovery, Postpartum (LDRP) charge nurse to assign to an LPN/VN? 1. Administering IV pain medication to a client three days postopertive cesarean section. 2. Drawing a trough vancomycin level on a client 3 days postpartum with bilaterial mastitis. 3. Reinforce how to perform perineal care to a primipara who is four hours postpartum. 4. Drawing routine admission labs on a client admitted in final stages of labor.

3. Correct: Client teaching may be reinforced by an LPN/VN on a stable client. 1. Incorrect: Administering IV pain medications is out of the scope of practice of LPN/VN. 2. Incorrect: Drawing lab work on a client with severe infection and only 3 days postpartum is an unstable client and needs care from the RN. 4. Incorrect: Drawing routine admission labs on a client in final stages of labor would be inappropriate because the client is potentially unstable and needs experienced LDRP nursing care.

What should the nurse include when providing teaching to a female client prescribed doxycycline for the treatment of acne? 1. Take this medication with food to maximize absorption. 2. Use a non-hormone method of birth control while taking this medication. 3. Wear protective clothing when outside. 4. Drink plenty of fluids while taking this medication. 5. Iron and calcium supplements can be taken with this medication.

2., 3., & 4. Correct: Doxycycline is a tetracycline antibiotic. Doxycycline can make birth control pills less effective. A non-hormone method of birth control (such as a condom, diaphragm, spermicide) should be used to prevent pregnancy while using doxycycline. Avoid exposure to sunlight or tanning beds. Doxycycline can make you sunburn more easily. Wear protective clothing and use sunscreen (SPF 30 or higher) when outdoors. Take doxycycline with a full glass of water. Drink plenty of liquids while taking this medicine. 1. Incorrect: Take on an empty stomach to maximize absorption, although may not be tolerated unless administered with food. 5. Incorrect: Do not take iron supplements, multivitamins, calcium supplements, antacids, or laxatives within 2 hours before or after taking doxycycline. Absorption will be altered. For instance, iron can bind to doxycycline in the gastrointestinal tract, which may prevent their absorption into the bloodstream and possibly reduce their effectiveness. To avoid or minimize the interaction, iron containing medications and doxycycline should preferably be taken at least three hours apart in most cases.

The nurse is planning health promotion strategies for an older client on a limited, fixed income who is trying to increase activity. The client has been cleared for moderate physical activity by the primary healthcare provider. Which strategies would be appropriate for this client? 1. Suggest that the client join a local gym for access to equipment and support. 2. Suggest contacting a neighbor so that they can walk each day in the neighborhood. 3. Encourage client to get up and walk around the house during each TV commercial break. 4. Suggest the client go to the community senior center for daily strengthening exercises. 5. Encourage client to use one-pound soup cans for muscle toning.

2., 3., 4. & 5. Correct: The neighborhood buddy is accessible and can be a source of emotional support too, which increases the likelihood of continuing the plan. This activity is easily accessible and burns calories during the day or evening. Senior centers usually do not cost any money for the client, and other seniors may help motivate the client to increase activity level. The use of ordinary items does not further strain a fixed income. 1. Incorrect: Joining a gym will require monthly fees, thus impacting financial resources in a negative way. Additionally, transportation to and from the gym could impact finances in a negative way.

The nurse recognizes that treatment has been successful in resolving fluid volume excess based on which assessment findings? 1. Continued lethargy 2. Heart rate 112/min 3. Decreasing shortness of breath 4. BP 114/78 5. Increased thirst

3. & 4. Correct: Urinary output should increase with decreasing shortness of breath as hydration is corrected, and BP should be normal. 1. Incorrect: Level of consciousness (LOC) should improve with perfusion to the brain. 2. Incorrect: Heart rate should decrease if hydration is corrected. 5. Incorrect: Thirst level should be decreased if hydration is corrected.

The nurse is passing morning medication on a busy medical-surgical unit and has been delayed in completing rounds. When re-evaluating how to distribute the remaining scheduled medications, which client would the nurse consider at greatest risk if medications are late? 1. The client with congestive heart failure receiving digoxin. 2. The client with epilepsy scheduled to receive phenytoin. 3. The client with myasthenia gravis on pyridostigmine. 4. The client with hypertension due for daily nifedipine.

3. Correct: Clients diagnosed with myasthenia gravis (MG) need a precise medication administration schedule since drugs such as pyridostigmine have a very short half-life. These medications are used to help decrease the weakness typical in MG clients, and the meds are only effective for 4 to 6 hours. It would be especially imperative to deliver the morning dose on time so that the client would not experience weakness during breakfast, increasing the risk for aspiration. 1. Incorrect: Digoxin is a cardiac glycoside which increases the contractility of the heart muscle while decreasing pulse rate, thus making the heart work more efficiently. This medication is generally given once daily and as such does not present dire consequences to the client if the dose is slightly later than usual. 2. Incorrect: The anti-seizure medication phenytoin is generally given once daily and has a very long half-life. Even if this medication is scheduled twice daily, the client is not likely to suffer any complications if the morning dose is late. 4. Incorrect: Nifedipine is a calcium channel blocker used to prevent daily angina episodes. Although these meds can be administered twice daily, this client is to receive just a daily dose and would not experience any complications from a slightly delayed morning dose.

Two days after a myocardial infarction, a client begins reporting orthopnea and dyspnea. Further assessment reveals bi-basilar crackles, jugular venous distension, an S3 heart sound, a BP of 100/60 mm Hg, and apical pulse of 90 beats per minute. The urine output has steadily declined over the past 12 hours. What should the nurse do first? 1. Notify the primary healthcare provider. 2. Increase the IV rate. 3. Elevate the head of the bed. 4. Observe for cardiac arrhythmias.

3. Correct: Elevate the head of the bed first. The client is reporting inability to breathe. (Orthopnea means the client needs to sit up to breathe better.) With ANY client having difficulty breathing, the first intervention for the nurse is to sit the client up. This client is showing s/s of heart failure. 1. Incorrect: Your next step is to call the primary healthcare provider after you do something to try to fix the problem. 2. Incorrect: Increasing the IV rate is contraindicated and would make the problem worse. 4. Incorrect: After an MI, all clients are observed for cardiac arrhythmias. This, however, does not fix the problem.

The home health nurse is caring for a client with a neurological urinary tract dysfunction. What information should be included when teaching the client how to perform intermittent self catheterization? 1. Performed in an emergency department (ED). 2. Prevents urinary catheter infections. 3. Perform as a clean procedure. 4. Requires using sterile gloves.

3. Correct: Home intermittent catheterization is a clean, not sterile technique when performed in the home environment. Home intermittent catheterization is preferred over continuous use of an indwelling catheter, as an indwelling catheter increases client risk of urinary tract infection (UTI). 1. Incorrect: The client can be taught to do self-catheterization at home. The client does not need to go to the emergency department (ED) to perform the self-catheterization procedure. 2. Incorrect: Performing intermittent self-catheterization at home is recommended for urinary retention. It does not prevent urinary tract infections. 4. Incorrect: Intermittent self-catheterization is a clean procedure, not sterile technique.

A full term infant is being assessed 12 hours after birth. The infant's respiratory rate is 50 and shallow, with periods of apnea. What action by the nurse takes priority? 1. Apply oxygen by mask at 1 liter. 2. Prepare for emergency intubation. 3. Continue monitoring every 15 minutes. 4. Notify the primary healthcare provider stat.

3. Correct: Normal respirations in the healthy neonate are generally shallow and expected to be between 30 and 50 times per minute with short periods of apnea up to 5 seconds. This infant is displaying a normal respiratory status for the newborn. The nurse should continue to monitor the infant. 1. Incorrect: This infant is showing normal adaptation to extrauterine life. The rate of 50, even with short periods of apnea, is within expected limits for a newborn. No need for oxygen at this time. 2. Incorrect:There is no indication that this infant is experiencing respiratory distress which would require intubation. Shallow respirations at the rate of 30 to 50 times per minute are expected, even with short apneic periods of 5 seconds. 4. Incorrect: There is no need to contact the primary healthcare provider. The respiratory status of this infant, even with short periods of apnea, is normal for a full term infant 12 hours after birth. Continued monitoring is all that is needed at this time.

The nurse is performing CPR on an adult client with facial and neck trauma. Following the administration of rescue breaths, where is the best location for the nurse to assess for a pulse in this client? 1. Apical area 2. Carotid artery 3. Femoral artery 4. Radial artery

3. Correct: Pulses that are best palpated are large and close to the trunk of the body. The femoral artery is large and at the trunk (proximal) of the body. 1. Incorrect: It would take too long to use a stethoscope and listen for an apical pulse on the client. A quicker area of checking the pulse would need to be used. 2. Incorrect: The client has bilateral neck trauma. The carotid artery would not be the best site to assess for a pulse. 4. Incorrect: The radial artery is not as large as the femoral artery and is distal to the femoral artery.

Which comment made by a client scheduled for a lumbar laminectomy and discectomy indicates to the nurse that the client needs further teaching? 1. After the incision is healed, I can go for daily walks. 2. By the time I am discharged, my back and leg pain will be better. 3. I can turn by myself after surgery, but I will need help to get out of bed. 4. The staff will frequently check my feet and legs for feeling and movement

3. Correct: The client must log roll with assistance. The spine must be kept in proper alignment to allow the area time to rest and heal. The nurse should reinforce this information with the client. 1. Incorrect: After the incision heals, it is acceptable practice to go for daily walks so this is an accurate understanding of what the client can do after the surgery. 2. Incorrect: Successful laminectomies and discectomies will relieve back and leg pain so this is accurate, also. 4. Incorrect: The nurse knows it is very important to perform neurovascular checks after ANY orthopedic surgery. Any changes from their baseline should be reported to the primary healthcare provider immediately. This indicates successful teaching.

The nurse is caring for a client following a transurethral resection of the prostate (TURP). The client has a 3 way irrigation catheter in place. Which observation would indicate the need to slow the irrigation? 1. Clots in urine 2. Bladder pressure 3. Clear urine 4. Bladder spasms

3. Correct: The irrigation is regulated so that the urine is free of clots and slightly pink tinged. When it becomes clear after surgery, the fluid is going too fast and not clearing any blood clots effectively. 1. Incorrect: The irrigation should be increased if you see clots in order to keep the catheter patent. 2. Incorrect: Bladder pressure may mean that the indwelling urinary catheter is obstructed. Either increase flow or manually irrigate catheter to ensure patency and no retention of fluid in the bladder. 4. Incorrect: Bladder spasms occur with clots so you do not want to slow the irrigation if this happens. This would indicate the need for increasing the irrigation fluid rate.

Following surgery, a client refuses to ambulate as prescribed. What action should the nurse take? 1. Notify the primary health care provider of client's refusal to ambulate. 2. Offer the client pain medication. 3. Explain complications associated with bed rest. 4. Perform passive range of motion exercises.

3. Correct: The nurse should educate the client about complications that can be prevented with ambulation, such as constipation, pneumonia, or deep vein thrombosis (DVT). 1. Incorrect: The first action should be for the nurse to educate the client so an informed decision can be made. This would put the nurse in an advocacy role and would more likely persuade the client to comply with ambulation. 2. Incorrect: The first action should be for the nurse to educate the client so that he/she can make an informed decision. Offering pain medication would be appropriate if pain is impeding the client's ability to move; however, pain medications may make the client at risk for falls so safety precautions would be priority. 4. Incorrect: The first action should be for the nurse to educate the client so that an informed decision can be made. Passive range of motion would not be the best option at this time.

One hour after administering pyridostigmine, the nurse notes increased salivation, lacrimation, and urination in the client. What initial action should the nurse take? 1. Administer a second dose of pyridostigmine. 2. Place client in side lying position. 3. Notify the primary healthcare provider. 4. Prepare for intubation and mechanical ventilation.

3. Correct: These are signs and symptoms of cholinergic crisis. The client can get increasingly worse. The primary healthcare provider can prescribe atropine as treatment of overdose. 1. Incorrect: Giving an additional dose of pyridostigmine will make the client worse. 2. Incorrect: For better respiratory effort the client should be placed in a semi fowler's position. 4. Incorrect: This can be done after notifying the primary healthcare provider.

A client has been prescribed vancomycin 1 gram IV every 12 hours for the treatment of methicillin-resistant staphylococcus aureus (MRSA). Which action by a new nurse when administering this medication would require intervention by the charge nurse? 1. Dilutes medication in NS 100 mL. 2. Delivers medication via an IV pump. 3. Calculates infusion rate at 30 minutes. 4. Monitors IV site every 30 minutes during infusion.

3. Correct: This dose of medication should be delivered over at least 60 minutes to prevent hypotension and ototoxicity. 1. Incorrect: The minimum dilution for 1 gram is 100 mL, so this action does not need intervention. 2. Incorrect: This is a correct action by the new nurse. A pump is required to ensure that medication is not delivered too rapidly. 4. Incorrect: A peripheral IV site should be monitored for pain, redness or swelling prior to initiating the infusion and every 30 minutes until the completion of the infusion.

A client with a history of congestive heart failure has an implantable cardioverter defibrillator (ICD) surgically implanted. What teaching points should the nurse provide the client prior to discharge? 1. Avoid hot baths and showers. 2. Increase intake of leafy green vegetable products. 3. Avoid magnets directly over the site. 4. Notify primary healthcare provider whenever a shock is delivered by the ICD. 5. Driving is not recommended for 1 year after placement of an ICD.

3., & 4. Correct: Magnets can deactivate the defibrillator. Other transmitter devices should also be avoided. Most arrhythmias need only one shock, but the healthcare provider should be notified when a shock is delivered so that monitoring can increase. 1. Incorrect: Hot baths or showers are not contraindicated with ICDs. 2. Incorrect: Increase of leafy green vegetable products would have no relation to the ICD but should be avoided if the client is on warfarin. 5. Incorrect: The client cannot drive for 6 months after implantation of an ICD and cannot drive for 6 months after any shock therapy from the ICD.

A client is being cared for on the orthopedic unit following a football game injury which resulted in a fracture of the left tibia and fibula. An open reduction of the fracture has been performed and a leg cast was applied. The client is receiving Morphine via a Patient Controlled Analgesia (PCA) pump at 2 mg/hr. The client begins reporting an increase in the pain level (9/10) that is not being relieved by the current Morphine dosing, and is experiencing a sensation that "pins are sticking" in the left foot. What action by the nurse is needed? 1. Increase the PCA dosing of Morphine. 2. Elevate the foot of the bed. 3. Perform neurovascular checks. 4. Apply ice around sides of cast. 5. Prepare for possible bivalving of the cast. 6. Notify primary healthcare provider.

3., 5., and 6. Correct: Are you recognizing what might be developing here? That's right! Compartment Syndrome! So what do you need to do? First, act quickly to assess the client by performing neurovascular checks. Remember the assessment of the 5 P's. If the neurovascular checks are poor, you will need to act promptly to restore circulation to the extremity. This may be accomplished by bivalving the cast to release compression on the extremity. The primary healthcare provider will also need to be notified so that the client can be taken to surgery quickly, if needed, for a fasciotomy to relieve the pressure in the leg area and restore circulation to the extremity below the affected area. Permanent nerve and vascular damage can occur quickly with compartment syndrome, so time is of the essence here! 1. Incorrect: The pain level is increasing instead of decreasing with the analgesia. Did you pick up on the paresthesia aspect being reported? This should prompt the nurse to act quickly and further assess the client. Administering more pain med would delay care and could possibly result in the client developing permanent neurovascular damage. 2. Incorrect: The circulation to the foot may be compromised. You would not want to elevate the extremity at this point. If the circulation is impaired with compartment syndrome, elevating the extremity could result in a further decrease in perfusion of the affected extremity rather than promoting circulation. This could cause a more rapid deterioration of the circulatory status and result in neurovascular damage to the extremity. 4. Incorrect: Applying ice around the sides of the cast is not going to relieve compartment syndrome. This would be delaying care and could possibly lead to neurovascular damage and a loss of the limb if the circulation is not restored.

Family members have been asking triage nurses if loved ones were admitted to the hospital during a national emergency situation with massive casualties. What response should be made by the nurses? 1. Tell the family members that information about clients cannot be provided. 2. Ask for the victims' permission before talking with the family members. 3. Instruct the family to wait for public announcements about victims. 4. Inform them if their family members have been admitted.

4. Correct. The national emergency situation allows waivers for the Health Insurance Portability and Accountability Act (HIPAA) provisions. Due to the emergency situation, the nurse may inform the family members about the status of their loved ones. 1. Incorrect. During a national disaster declared by the President, information may be given to families without client consent. 2. Incorrect. There is no need to make the family more worried if information is known. Waivers for certain elements of HIPAA are allowed during the emergency period. 3. Incorrect. The nurse may legally give information to the family. The triage nurse may provide information concerning their family members.

A hysterical college student arrives in the emergency department in bloody soiled clothing. The nursing assessment reveals facial bruising and multiple contusions consistent with the client's report of being raped. Which initial nursing intervention takes priority at this time? 1. Notify police of the alleged rape. 2. Allow the client privacy to wash self. 3. Remove clothing and bag for evidence. 4. Encourage client to express fears and anxiety.

4. Correct: Although there are specific protocols that must be followed when dealing with rape clients, it is important to remember that this client had all control taken away during the attack. The emotional effects of rape are as traumatizing as the physical injuries. Allowing the client to first express emotions, such as fear or anxiety, returns a small amount of control to a situation in which the client has little or none. 1. Incorrect: The police will indeed be notified of the situation by the hospital personnel, but the nurse's initial priority is to focus on the client's needs, both physical and emotional. 2. Incorrect: The client will not be permitted to clean self until all evidence has been collected per protocol. However, initial contact between nurse and client should focus on more than just the physical aspects of the situation. 3. Incorrect: Collection of all evidence for the police is a crucial part of treating rape clients and will be completed according to protocols. But it is more important to remember that this client has already been violated during the attack. Removing clothing before addressing emotional needs may further exacerbate that sense of violation.

The primary healthcare provider prescribes glycopyrrolate 0.2 mg IM thirty minutes prior to electroconvulsive therapy (ECT). What should be the nurse's response when the client asks why this drug is being given? 1. "The action of the medication is complex." 2. "This drug will prevent you from having a seizure." 3. "This medication will relax your muscles so that you do not break a bone." 4. "Glycopyrrolate will decrease stomach secretions."

4. Correct: Glycopyrrolate is an anticholinergic. Glycopyrrolate blocks the activity of acetylcholine which reduces secretions in the mouth, throat, airway, and stomach. It is used prior to procedures to decrease the risk of aspiration. 1. Incorrect: The client has a right to be told the reason the drug is given. This is a nontherapeutic communication response. The nurse should not refuse the client's desire to understand their medications. 2. Incorrect: Glycopyrrolate blocks the secretions in the mouth, throat, airway and stomach. The medication does not prevent the client having a seizure. The ECT will induce a seizure, which is the desire. 3. Incorrect: This is not the drug's purpose so this would be incorrect information to give to the client.

A nurse attaches a client to continuous cardiac monitoring due to a potassium level of 2.8 mEq (2.8 mmol). The nurse should monitor for which dysrhythmia? 1. Third degree heart block 2. Atrial fibrillation 3. Premature atrial contractions 4. Premature ventricular contractions

4. Correct: Hypokalemia is reflected by the EKG. The earliest EKG change is often premature ventricular contractions (PVCs) which can deteriorate into ventricular tachycardia or fibrillation (VT/VF) without appropriate potassium replacement. 1. Incorrect: Atrial-ventricular blocks are not often seen initially with hypokalemia. 2. Incorrect: Atrial fibrillation is not often seen with hypokalemia. 3. Incorrect: PACs are not often seen initially with hypokalemia.

The edrophonium (Tensilon) test has been prescribed for a client. Which statement by the client would indicate to the nurse that the client understands this test? 1. "This medication will be given to me as an IM injection immediately after my muscles are tired." 2. "This test will determine if I have multiple sclerosis." 3. "The test is positive if my muscles do not get stronger after injection with this medication." 4. "I will be asked to perform a repetitive movement to test my muscles."

4. Correct: If the primary healthcare provider suspects myasthenia gravis (MG), the client will be asked to perform a repetitive movement to test a group of muscles. 1. Incorrect: The medication is given IVP after the muscle group has become fatigued. 2. Incorrect: The edrophonium (Tensilon) test is used to diagnose myasthenia gravis. 3. Incorrect: A person tests positive for MG if their muscles get stronger after being injected with edrophonium (Tensilon).

During morning report, the nurse learns that a client's call bell is not working and maintenance cannot do repairs until tomorrow. The nurse is aware that the safest temporary method for the client to signal staff is what? 1. Provide a hand-held bell for client to ring. 2. Ask family to stay with client to alert staff. 3. Tell client to call out loudly to the staff. 4. Have staff visit client's room every 15 minutes.

4. Correct: It is vital for clients to be able to contact or alert staff for needs and concerns. The safest method is for the staff to check on the client at specified intervals. This will help alleviate client concerns about being able to signal the staff while ensuring that someone actually observes the client. 1. Incorrect: While a hand-held bell could be an option, it is not reliable. The client could easily push it onto the floor, or it could become tangled in the linens. Additionally, depending on the noise level of the unit, a hand bell could either disturb other clients or not be heard by staff. 2. Incorect: It is not the responsibility of the family to sit with the client 24/7 just because the hospital has non-working equipment. Not only would this be an imposition, it violates most visiting policies and places the burden on the family. 3. Incorrect: Having a client call out to staff is both inefficient and unsafe. Assuming the client's voice is even loud enough to be heard, it is unlikely that the verbalizations of one client could be distinguished from others that may call out because of dementia or normal nighttime utterances. This is not safe.

The nurse assesses a multigravida who is four hours postpartum. Findings include that fundus is firm, 1 centimeter above the umbilicus, and deviated to the right side. The lochia is moderately heavy and bright red. Which nursing intervention has priority? 1. Massage the fundus. 2. Administer intravenous oxytocin. 3. Document these normal findings. 4. Assist the client up to void.

4. Correct: These findings are caused by a full bladder, which prevents the uterus from contracting down and achieving homeostasis. Once the bladder is empty, the fundus will contract adequately and return to its normal location at level of umbilicus or 1 finger breadth below the umbilicus and in the midline. A distended bladder will displace the uterus, usually to the right. 1. Incorrect: The nurse may check fundus after client voids to ensure that this fixes the problem. 2. Incorrect: Administering oxytocin is not the first intervention for this issue. 3. Incorrect: These are not normal findings so this would be incorrect information for the nurse to document.

The nurse is educating a client on diabetes management. The client is asking questions that cause the nurse to be concerned about the client's ability to retain the information. Which of the following would be the best technique for the nurse to use to enhance the retention of information by the client? Provide the client with a thorough reference list. Conduct the education using a lecture format. Speak slowly to allow information to be absorbed. Repeat important information during the presentation.

Repetition is an effective means of reinforcing critical information and enhancing content retention. The other options will not increase the client's ability to retain information and may decrease the client's concentration and ability to retain critical information.

The client is discussing the client's medication history with the nurse. During the discussion, the client pulls out a list of the prescribed medications which include fish oil and St. John's Wort. Which is the nurse's understanding of why these alternative therapies are used by the client? The client has a history of depression. The client has a history of coronary diseases. The client has a history of digestive issues. The client has a history of diabetes.

The client has a history of depression. Fish oil and St. John's wort are alternative therapies to treat depression.

A client is treated in the emergency department for acute muscle strain in the left leg caused by trying a new exercise. During discharge preparation, the nurse should provide which instruction? "Apply ice packs for the first 12 to 18 hours." "Apply heat packs for the first 24 to 48 hours." "Apply ice packs for the first 24 to 48 hours, then apply heat packs." "Apply heat packs for the first 24 hours, then apply ice packs for the next 48 hours."

The nurse should instruct the client to apply ice packs to the injured area for the first 24 to 48 hours to reduce swelling and then apply heat to increase comfort, promote reabsorption of blood and fluid, and speed healing. Applying ice for only 12 to 18 hours may not keep swelling from recurring. Applying heat for the first 24 to 48 hours would worsen, not ease, swelling. Applying ice 48 hours after the injury would be less effective because swelling already has occurred by that time.

A client comes to the emergency department with symptoms of chest pain radiating down the left arm, dyspnea, and diaphoresis. An electrocardiogram (EKG) shows ST segment elevation and the client is diagnosed with an ST segment-elevation myocardial infarction (STEMI). In order to determine if the client is a candidate for thrombolytic therapy, which question should the nurse ask? "What time did your chest pain start?" "Did you take any nitroglycerine before coming to the emergency department?" "Do you have any allergies?" "Is this the first time you experienced this type of pain?"

Thrombolytic therapy must be started within 6 hours of the onset of the myocardial infarction (MI). The time the chest pain started is the priority. The nurse can assess for allergies once the time is determined. Nitroglycerine will not impact the administration of thrombolytic therapy.

A client uses timolol maleate eye drops. What is the expected outcome of this drug? constricting the pupils dilating the canals of Schlemm reducing aqueous humor formation improving the ability of the ciliary muscle to contract

Timolol maleate is commonly administered to control glaucoma. The drug's action is not completely understood, but it is believed to reduce aqueous humor formation, thereby reducing intraocular pressure. Timolol does not constrict the pupils; miotics are used for pupillary constriction and contraction of the ciliary muscle. Timolol does not dilate the canal of Schlemm.


Ensembles d'études connexes

ACCT 302 - Shareholders' Equity (Ch 18)

View Set

Chapter 1: Personal Financial Planning in Action

View Set

Digestive Enzyme: Pancreatic Amylase

View Set

10 Plant Evolution and Diversity

View Set